ABFM Bank Flashcards

1
Q

compare influenza to URI, mycoplasma pneumonia, legionnaires disease

A

Influenza - fever, nonproductive cough, prostrating illness
URI - not so prostrating illness
mycoplasma pneumonia - milder illness, longer incubation period than influenza
legionnaires disease - mild illness to life threatening pneumonia, can cause point source epidemics, longer incubation period, diarrhea prominent sx

How well did you know this?
1
Not at all
2
3
4
5
Perfectly
2
Q

compare Henoch-Schonlein purpura to other vasculitis, Takayasu arteritis, Wegener granulomatosis, Polyarteritis nodosa

A

Henoch-Schonlein purpura - IgA-mediated small-vessel vasculitis. Triad of nonthrombocytopenic palpable purpura, colicky abdominal pain, and arthritis. Also has conjunctival injection, mucosal erythema, rash, and lymphadenopathy

Takayasu arteritis - night sweats, fatigue, weight loss, myalgia, and arthritis. Later findings hypertension, skin lesions, and cardiac disorders

Wegener granulomatosis - weight loss and fatigue, later findings including respiratory problems, ophthalmologic lesions, neuropathies, glomerulonephritis, and skin lesions.

Polyarteritis nodosa - fatigue, fever, and myalgias. Skin lesions, gastrointestinal symptoms such as postprandial abdominal pain, and cardiac lesions.

How well did you know this?
1
Not at all
2
3
4
5
Perfectly
3
Q

substances reported to cause false-positive urine drug screens include what

A

poppy seed - can produce amounts of morphine and codeine

selegiline, Vicks inhalers, NSAIDs, oxaprozin, fluoroquinolones, rifampin, venlafaxine, and dextromethorphan.

How well did you know this?
1
Not at all
2
3
4
5
Perfectly
4
Q

What is the Ottawa ankle rules?

A

To determine if ankle or foot XR needed.

Patients who were able to bear weight immediately following their injury and who can take 4 steps
independently in a clinical setting require radiographic study only when the following criteria are met: pain is present in the malleolar zone and bony tenderness of the posterior edge or tip of either malleolus is elicited (ankle radiograph), or pain is present in the midfoot zone and bony tenderness of either the base of the fifth metatarsal or the navicular region is

https://www.google.com/url?sa=i&url=http%3A%2F%2Fwww.emdocs.net%2Fankle-x-rays%2F&psig=AOvVaw0bVz9NdEcvrhSgniemBcoI&ust=1639453857811000&source=images&cd=vfe&ved=0CAsQjRxqFwoTCIDggtrv3_QCFQAAAAAdAAAAABAJ

How well did you know this?
1
Not at all
2
3
4
5
Perfectly
5
Q

When can child return to school once he/she starts treatment for bacterial conjunctivitis?

A

Once therapy is initiated, children with bacterial conjunctivitis should be allowed to remain in school. Careful hand hygiene is important and behavior must be appropriate to maintain adequate hygiene. No specific length of treatment or evidence of clinical response is required before returning to school.

How well did you know this?
1
Not at all
2
3
4
5
Perfectly
6
Q

Which site of head and neck lymphadenopathy is most likely malignant?

A

supraclavicular

How well did you know this?
1
Not at all
2
3
4
5
Perfectly
7
Q

You give Rhogam to Rh negative or positive mother? and how much?

A

RHO immune globulin (RhoGAM) is recommended in any Rh-negative patient who is unsensitized with fetomaternal hemorrhage. For gestational age is 12 weeks or less, give 50 mcg of RhoGAM. For gestational age is greater than 12 weeks, give 300 µg of RhoGAM.

How well did you know this?
1
Not at all
2
3
4
5
Perfectly
8
Q

what is the Rh antigen?

A

Rh negative means the person does not have type D antigen on their red blood cells. If a woman is exposed to Rh D antigen–positive red blood cells, she can have an immune response of variable strength. This may occur in the setting of pregnancy (transplacental fetomaternal transfusion), or exposure outside of pregnancy (e.g., transfusion with mismatched blood). An antibody titer of 1:4 poses much less risk to the fetus than a titer of 1:64.

How well did you know this?
1
Not at all
2
3
4
5
Perfectly
9
Q

What complications occurs most frequently after Roux-en-Y gastric bypass surgery for obesity?

A

Iron and vitamin B12 deficiencies

How well did you know this?
1
Not at all
2
3
4
5
Perfectly
10
Q

what the of anemia is most common in infancy

A

iron deficiency anemia

How well did you know this?
1
Not at all
2
3
4
5
Perfectly
11
Q

The scabies mite is predominantly transmitted by

A

direct personal contact

How well did you know this?
1
Not at all
2
3
4
5
Perfectly
12
Q

Electrosurgical destruction treatment is contraindicated for

A

use of a pacemaker and the treatment of melanoma

How well did you know this?
1
Not at all
2
3
4
5
Perfectly
13
Q

Clinical picture of a when Crohn’s disease affects primarily the distal small intestine (regional enteritis)

A

A young person, usually in the second or third decade, will present with a period of episodic abdominal pain, largely postprandial and often periumbilical, occasionally with low-grade fever and mild diarrhea. Anorexia, nausea, and vomiting may be present. Weight loss frequent. Some patients may have tenderness in RLQ and even of a palpable mass in RLQ.

How well did you know this?
1
Not at all
2
3
4
5
Perfectly
14
Q

When family dynamics lead to conflict during an office visit, it is best for the physician to attempt to remain neutral by avoiding triangulation which is?

A

When the two sides in conflict each attempt to align with a third party. Priority should be given to the patient’s right to privacy and confidentiality, and the physician should ask permission from the patient to discuss his or her health issues with other people. Physicians should always remember who they are primarily responsible to.

How well did you know this?
1
Not at all
2
3
4
5
Perfectly
15
Q

What is DRESS?

A

DRESS is acronym for Drug Reaction with Eosinophilia and Systemic Symptoms. Hallmark of DRESS syndrome is erythroderma accompanied by fever, LA, elevated LFTs, and eosinophilia. The offending medication should be discontinued immediately and treatment with corticosteroids should be initiated. Seizure medications such as carbamazepine, phenytoin, lamotrigine, and phenobarbital are responsible for approximately one-third of cases. Allopurinol-associated DRESS syndrome has the highest mortality rate.

How well did you know this?
1
Not at all
2
3
4
5
Perfectly
16
Q

Described celiac disease (gluten-sensitive enteropathy)

A

Approximately 7% of type 1 diabetics have celiac disease. A number of other autoimmune syndromes have been associated with celiac disease, including thyroid disease and rheumatoid arthritis. There is no reported association with type 2 diabetes. Gastrointestinal involvement may include diarrhea, constipation, or other symptoms of malabsorption, such as bloating, flatus, or belching. Fatigue, depression, fibromyalgia-like symptoms, aphthous stomatitis, bone pain, dyspepsia, gastroesophageal reflux, and other nonspecific symptoms may be present and can make the diagnosis quite challenging. Dermatitis herpetiformis is seen in 10% of patients with celiac disease. Serum antibody testing, especially IgA antiendomysial antibody, is highly sensitive and specific. Definitive diagnosis generally requires EGD with biopsy of the distal duodenum to detect characteristic villous flattening.

How well did you know this?
1
Not at all
2
3
4
5
Perfectly
17
Q

What is secondary hyperaldosteronism

A

sodium retention and thus decreased urinary sodium excretion, while potassium secretion is normal to increased.

How well did you know this?
1
Not at all
2
3
4
5
Perfectly
18
Q

What imaging modalities would be most helpful for confirming a diagnosis of appendicitis?

A

spiral CT.
Ultrasonography is used in women who are pregnant and women in whom there is a high degree of suspicion of gynecologic disease.

How well did you know this?
1
Not at all
2
3
4
5
Perfectly
19
Q

Which drug can be used as part of a test for diagnosing renovascular hypertension, but would also increase the risk for azotemia if used for treatment?

A

ACE inhibitors can significantly worsen renal failure in patients with hypertension caused by renovascular disease. Hyperkalemia is an associated problem. Captopril renography is a useful diagnostic screening test.

How well did you know this?
1
Not at all
2
3
4
5
Perfectly
20
Q

Patients with symptomatic heart failure associated with a reduced systolic ejection fraction or left ventricular remodeling should be initially treated with which one of the following agents?

A

CHF patients treated with ACE inhibitors survive longer, and all such patients should take ACEi if tolerated.

How well did you know this?
1
Not at all
2
3
4
5
Perfectly
21
Q

Adverse effects of these drugs.
Tumor necrosis factor (TNF) inhibitors
Amiodarone

A
TNF inhibitors (adalimumab) are currently approved by FDA for the treatment of rheumatic diseases such as rheumatoid arthritis, ankylosing spondylitis,
psoriatic arthritis, and juvenile idiopathic arthritis. All drugs in this class carry an FDA black-box warning for potentially developing primary tuberculosis or reactivating latent tuberculosis.

Amiodarone is associated with a subacute cough and progressive dyspnea due to pulmonary toxicity.

How well did you know this?
1
Not at all
2
3
4
5
Perfectly
22
Q

How to switch a patient on dabigatran (direct thrombin inhibitor) to warfarin?

A

Start warfarin 3 days prior to stopping dabigatran. Bridging with a parenteral agent is not necessary. Dabigatran is known to increase the INR, so the INR will not reflect warfarin’s effect until dabigatran has been withheld for at least 2 days.

How well did you know this?
1
Not at all
2
3
4
5
Perfectly
23
Q
Mechanism of action of diabetes meds?
Metformin
Thiazolidinediones (pioglitazone)
Sulfonylureas
Meglitinides
A

Metformin - main effect on serum glucose results from inhibition of gluconeogenesis in the liver

Thiazolidinediones (pioglitazone) - sensitize peripheral tissues to insulin

Sulfonylureas (glipizide) and meglitinides (Repaglinide and nateglinide)- stimulate insulin release from the pancreas

How well did you know this?
1
Not at all
2
3
4
5
Perfectly
24
Q

Epidemics of gastroenteritis on cruise ships are consistent with

A

Norovirus infections due to waterborne or foodborne spread. Common cause of waterborne epidemics of gastroenteritis, and have been shown to be responsible for outbreaks in nursing homes, on cruise ships, at summer camps, and in schools. Symptomatic treatment.

How well did you know this?
1
Not at all
2
3
4
5
Perfectly
25
Q

What is treatment for primary hyperparathyroidism for patients <50 years old or symptomatic patients, such as those with a fragility fracture?

A

Parathyroidectomy. If a patient is older, is a poor surgical candidate, or has asymptomatic disease, long-term monitoring with treatment focused on reducing bony complications can be considered

How well did you know this?
1
Not at all
2
3
4
5
Perfectly
26
Q

Venous thrombosis is associated with what conditions

A

activated protein C resistance (factor V Leiden), trauma, surgery, malignancy, sepsis, and oral contraceptive use.

How well did you know this?
1
Not at all
2
3
4
5
Perfectly
27
Q

agents that can induce hypokalemia include

A

β-Agonists (albuterol), pseudoephedrine, insulin, thiazides diuretics

How well did you know this?
1
Not at all
2
3
4
5
Perfectly
28
Q

agents that can induce hypokalemia include

A

β-Agonists (albuterol), pseudoephedrine, insulin, thiazides diuretics

How well did you know this?
1
Not at all
2
3
4
5
Perfectly
29
Q

what is treatment for infantile gonococcal infection

A

A single dose of 25–50 mg/kg of ceftriaxone IV or IM. Ophthalmia neonatorum and sepsis are the most severe gonococcal infections in newborns

How well did you know this?
1
Not at all
2
3
4
5
Perfectly
30
Q

which nasal medication has beneficial effect for symptoms of the common cold in an adult?

A

Ipratropium (Atrovent) nasal spray is the only nasally inhaled anticholinergic that decreases rhinorrhea and sneezing in the common cold.

How well did you know this?
1
Not at all
2
3
4
5
Perfectly
31
Q

Which physical examination findings is pathognomonic for slipped
capital femoral epiphysis?

A

Slipped capital femoral epiphysis (SCFE) typically occurs in young adolescents during the growth spurt, when the femoral head is displaced posteriorly through the growth plate. Physical activity, obesity, and male sex are predisposing factors. There is pain with physical activity, most commonly in the upper thigh anteriorly, but one-third of patients present with referred lower
thigh or knee pain. Hallmark of SCFE exam is limited internal rotation of the hip. Specific to SCFE is the even greater limitation of internal rotation when the hip is flexed to 90°.

How well did you know this?
1
Not at all
2
3
4
5
Perfectly
32
Q

What is the usual reservoir for hantavirus?

A

Virus is spread by inhalation of dried aerosolized deer mouse urine or droppings

How well did you know this?
1
Not at all
2
3
4
5
Perfectly
33
Q

what is the sensitivity, specificity, NPV, and PPV chart?

A

draw it

How well did you know this?
1
Not at all
2
3
4
5
Perfectly
34
Q

Which cardiac rhythm abnormalities is most common in patients with anorexia nervosa?

A

Sinus bradycardia is almost universally present in patients with anorexia nervosa. Hypothesized that this is due to vagal hyperactivity

How well did you know this?
1
Not at all
2
3
4
5
Perfectly
35
Q

A nucleic acid amplification test is performed on a urine sample, and the results are positive for gonorrhea and negative for Chlamydia. Which one of the following is the recommended treatment for this patient?

A

Ceftriaxone, 250 mg intramuscularly, plus azithromycin (Zithromax), 1 g orally. High incidence of coinfection with Chlamydia, even if testing is negative

How well did you know this?
1
Not at all
2
3
4
5
Perfectly
36
Q

Polypharmacy is common in the elderly population. Older antihistamines such as in acetaminophen/diphenhydramine can cause what ADE in elderly?

A

CNS effects such as cognitive slowing and delirium. Anticholinergic properties of older antihistamines include dry mouth, constipation, blurred vision, drowsiness, and sedation

How well did you know this?
1
Not at all
2
3
4
5
Perfectly
37
Q

What are facial dysmorphic features of fragile X syndrome and Prader-Willi syndrome?

A

Fragile X syndrome - In prepubescent male child, have large ears, an elongated face, macrocephaly, or frontal bossing. After puberty, a prominent jaw and macro-orchidism.

Prader-Willi syndrome - narrow distance between the temples, almond-shaped eyes, and a thin upper lip.

How well did you know this?
1
Not at all
2
3
4
5
Perfectly
38
Q

how do phosphate binders work to hypercalcemia in secondary hyperparathyroidism of ESRD?

A

Phosphate binders (sevelamer, calcium acetate. Decreasing serum phosphate lowers the feedback stimulation of parathyroid hormone secretion by the parathyroid gland. Normalizing parathyroid levels improves serum calcium levels.

How well did you know this?
1
Not at all
2
3
4
5
Perfectly
39
Q

11-year-old male is brought to your office for evaluation of bilateral posterior heel pain that has occurred for the past few months. No swelling or tenderness of the heel or Achilles tendon. What is condition?

A

Calcaneal apophysitis (Sever disease) is the most common etiology of heel pain in children, usually occurring between 5 and 11 years old. Bones grow faster than the muscles and tendons. A tight Achilles tendon then pulls on its insertion site at the posterior calcaneus with repetitive running or jumping activities, causing microtrauma to the area. There may be swelling and tenderness in this area and passive dorsiflexion may increase the pain. Treatment involves decreasing pain-inducing activities, anti-inflammatory or analgesic medication if needed, ice, stretching and strengthening of the gastrocnemius-soleus complex, and the use of orthotic devices.

How well did you know this?
1
Not at all
2
3
4
5
Perfectly
40
Q

Differentiate chlamydial pneumonia, Staphylococcal pneumonia, RSV?

A

Chlamydial pneumonia is usually seen in infants 3–16 weeks of age, and these patients frequently have been sick for several weeks (subacute illness). The infant appears nontoxic, afebrile, tachypneic, prominent cough. PE with diffuse crackles with few wheezes, and conjunctivitis is present in about 50% of cases. CXR will show hyperinflation and diffuse interstitial or patchy infiltrates. There may be eosinophilia.

RSV infections start with rhinorrhea and pharyngitis, followed in 1–3 days by a cough and wheezing. Auscultation of the lungs will reveal diffuse rhonchi, fine crackles, and wheezes. CXR is often normal. If the illness progresses, coughing and wheezing increase, air hunger and intercostal retractions develop, and evidence of hyperexpansion of the chest is seen. In some infants the course of the illness may be similar to that of pneumonia. Rash or conjunctivitis may occur occasionally, and fever is an inconsistent sign. The WBC count will be normal or elevated.

Staphylococcal pneumonia has a sudden onset. Infant appears very ill and has a fever, and initially may have an expiratory wheeze simulating bronchiolitis. Signs of abdominal distress, tachypnea, dyspnea, and localized or diffuse bronchopneumonia or lobar disease may be present. WBC count will show a prominent leukocytosis.

How well did you know this?
1
Not at all
2
3
4
5
Perfectly
41
Q

treatment for acute hyperthyroidism

A

B-blocker and Methimazole first.

Propylthiouracil (PTU) can cause hepatocellular damage so don’t use. Then radioactive iodine.

How well did you know this?
1
Not at all
2
3
4
5
Perfectly
42
Q

Symptoms and treatment for epiglottis?

A

Sx: Hoarseness, dysphagia, stridor, drooling, fever, chills, and respiratory distress
Tx: IV antibiotic should be given immediately, ideally with a B-lactam drug that exhibits activity against MRSA

How well did you know this?
1
Not at all
2
3
4
5
Perfectly
43
Q

Signs of streptococcal pharyngitis

A

Fever over 100.4°F, tonsillar exudates, anterior cervical lymphadenopathy, and absence of cough.

How well did you know this?
1
Not at all
2
3
4
5
Perfectly
44
Q

what pain medication is CI in liver cirrhosis?

A

NSAIDs due to the risk of precipitating hepatorenal syndrome

How well did you know this?
1
Not at all
2
3
4
5
Perfectly
45
Q

What is erythrasma

A

Infection with Corynebacterium minutissimum
Coral-red fluorescence on Wood’s light examination
Erythromycin is treatment

How well did you know this?
1
Not at all
2
3
4
5
Perfectly
46
Q

which diabetes drug is the most likely to cause hypoglycemia

A

Sulfonylureas: Glyburide is more likely to cause hypoglycemia than Glipizide. Glyburide should rarely be used in the elderly.

How well did you know this?
1
Not at all
2
3
4
5
Perfectly
47
Q

what is calcitonin-salmon (Miacalcin) used for?

A

Vertebral fracture risk reduction (labeled use) or osteoporotic vertebral fracture pain relief (off label use)

How well did you know this?
1
Not at all
2
3
4
5
Perfectly
48
Q

Extracorporeal shock wave therapy is effective for what condition?

A

Calcific tendinitis of the rotator cuff.

How well did you know this?
1
Not at all
2
3
4
5
Perfectly
49
Q

What is first-line therapy for hypertension in pregnancy?

A

Labetalol
CI in pregnancy: ACEi, ARB, metoprolol due to adverse effects
Metoprolol: pregnancymetoprolol with fetal growth restriction birth defects and ACEi, ARB: risk of birth defects and fetal or neonatal renal failure

How well did you know this?
1
Not at all
2
3
4
5
Perfectly
50
Q

What is criteria for hypoactive sexual desire disorder (HSDD) and treatment?

A

Diagnosis includes two components:
(1) recurrent deficiency or absence of sexual desire or receptivity to sexual activity, and (2) distress about such a deficiency.

Tx: Topical testosterone, in either patch or gel form, has shown consistent improvements in arousal, desire, fantasy, orgasm, and overall satisfaction in cases of hypoactive sexual desire disorder (HSDD)

How well did you know this?
1
Not at all
2
3
4
5
Perfectly
51
Q

what is treatment for thrombosed external hemorrhoid, internal hemorrhoid, or hemorrhoids not in acute stage?

A

Elliptical excision of the thrombosed external hemorrhoid and overlying skin under local anesthesia.

Rubber band ligation and infrared coagulation are options for internal hemorrhoids.

Sitz baths, bulk laxatives, stool softeners, and local analgesia may be helpful. Avoid precipitating factors such as prolonged standing/sitting, constipation, and delay of defecation.

How well did you know this?
1
Not at all
2
3
4
5
Perfectly
52
Q

What class of psych meds have shown beneficial effect on the symptoms of irritable bowel syndrome (IBS)?

A

SSRIs and tricyclic antidepressants

How well did you know this?
1
Not at all
2
3
4
5
Perfectly
53
Q

What is screening guideline for diabetes mellitus

A

Test for DM in adults who have a BMI ≥25 kg/m2 and have one or more risk factors such as physical inactivity, a first degree relative with diabetes, a high-risk ethnicity, hypertension, hyperlipidemia, or polycystic ovary syndrome.

In patients with no risk factors, test at age 45.

How well did you know this?
1
Not at all
2
3
4
5
Perfectly
54
Q

Adverse effect of aromatase inhibitor such as letrozole (Femara) and tamoxifen (Soltamox)?

A

letrozole (Femara): Myalgias and noninflammatory arthralgias

tamoxifen (Soltamox): Endometrial cancer

How well did you know this?
1
Not at all
2
3
4
5
Perfectly
55
Q

what isMyelodysplastic syndrome

A

Hematologic malignancy with a predisposition to leukemic transformation. It can present with anemia, thrombocytopenia, neutropenia, or any combination of these. Anemia occurs in 80%–85% of patients and is typically macrocytic

How well did you know this?
1
Not at all
2
3
4
5
Perfectly
56
Q

What are the asthma classifications?

A

Intermittent: Symptoms less than or equal to twice weekly, nighttime awakenings ≤2 times/month, short-acting β-agonist usage ≤2 days/week, no interference with daily activities, and normal FEV1 and FEV1/FVC ratio at baseline

Mild Persistent: Symptoms >2 days/week but not daily, nighttime awakenings 3–4 times/month, short-acting β-agonist usage >2 days/week but not more than once daily, minor limitation to daily activities, FEV1 ≥80% predicted, and normal FEV1/FVC ratio

Moderate Persistent: Daily symptoms, nighttime awakenings greater than once weekly but not nightly, daily use of a short-acting β-agonist, some limitation to daily activity, FEV1 >60% but <80% of predicted, and FEV1/FVC ratio reduced by 5%

Severe Persistent: Symptoms throughout the day, nighttime awakenings nightly, short-acting β-agonist usage several times daily, extremely limited daily activities, FEV1 <60% of predicted, and FEV1/FVC ratio reduced by >5%
Status asthmaticus is a medical emergency and requires emergent treatment in a hospital setting.

How well did you know this?
1
Not at all
2
3
4
5
Perfectly
57
Q

Treatment for mallet fracture (finger)?

A

mallet fracture: caused by load to the tip of an extended finger that causes forced flexion at the distal interphalangeal (DIP) joint and leads to dorsal extensor tendon rupture or avulsion fracture (tendon and piece of bone attached).
Tx: splint the finger DIP joint in extension for 8 weeks

How well did you know this?
1
Not at all
2
3
4
5
Perfectly
58
Q

what can you give a patient with end stage cancer with few weeks life expectancy and becomes very depressed?

A

Psychostimulants (i.e. methylphenidate) reduce symptoms of depression within days

How well did you know this?
1
Not at all
2
3
4
5
Perfectly
59
Q

signs of heart failure

A

HF: Fluid overload (edema), JVD, a third heart sound, an elevated BNP level, and a responds to diuretics.

A BNP level <100 pg/mL can help rule out heart failure.

How well did you know this?
1
Not at all
2
3
4
5
Perfectly
60
Q

Treatment for Traveler’s diarrhea

A

Most common cause of traveler’s diarrhea is enterotoxigenic Escherichia coli. Fluoroquinolones such as ciprofloxacin have been shown to significantly reduce the duration and severity of traveler’s diarrhea when given for 1–3 days

How well did you know this?
1
Not at all
2
3
4
5
Perfectly
61
Q

Treatment for prevention of constipation in opioid-induced constipation

A

Constipation is a very common side effect of opioids that does not resolve with time. Start an appropriate bowel regimen with the initiation of opioid therapy to prevent constipation.

Polyethylene glycol, lactulose, magnesium hydroxide, and senna with docusate

How well did you know this?
1
Not at all
2
3
4
5
Perfectly
62
Q

Which activities are most likely to be impaired in early dementia?

A

Basic activities of daily living, such as dressing, eating, toileting, and grooming, are generally intact in early dementia. Instrumental activities of daily living, such as managing money and medications, shopping, cooking, housekeeping, and transportation, which often require calculation or planning, are frequently impaired in early dementia.

How well did you know this?
1
Not at all
2
3
4
5
Perfectly
63
Q
Adverse effect of these psych meds:
Carbamazepine
Aripiprazole and olanzapine 
Lithium 
Imipramine
A

Carbamazepine (Anticonvulsant) - agranulocytosis
Aripiprazole and olanzapine (2nd generation atypical antipsychotics) - black box warnings for an increased risk of death in the elderly.
Lithium (Antimanic - Bipolar d/o, MDD unipolar) - lithium toxicity and thyroid dysfunction
Imipramine (TCA) - cardiac toxicity, EKG monitoring is recommended

How well did you know this?
1
Not at all
2
3
4
5
Perfectly
64
Q

These tests for what injury?
Lachman test
Posterior drawer test
McMurray and Thessaly

A

Lachman test - anterior cruciate ligament
Posterior drawer test - posterior cruciate ligament
McMurray and Thessaly - meniscal tears

How well did you know this?
1
Not at all
2
3
4
5
Perfectly
65
Q

Patients with celiac disease are at increased risk of?

A

Celiac disease - increased risk of osteoporosis due to bone loss from
decreased calcium and vitamin D absorption.

How well did you know this?
1
Not at all
2
3
4
5
Perfectly
66
Q

How many days before surgery before surgery should Clopidogrel be stopped?

A

Patients receiving dual antiplatelet therapy who require bypass surgery should continue taking aspirin. Clopidogrel or prasugrel should be stopped 5 days before the surgery.

How well did you know this?
1
Not at all
2
3
4
5
Perfectly
67
Q

Pulmonary hypertension can be due to various causes (left heart failure, chronic thromboembolic disease, etc). When due to left heart failure, you want to do what?

A

maximize treatment for her heart failure and other comorbidities.

How well did you know this?
1
Not at all
2
3
4
5
Perfectly
68
Q

Treatment for methanol poisoning

A

Depending on its severity, give ethanol to inhibit the metabolism of methanol to toxic metabolites, hemodialysis to remove alcohol and its toxins, and vigorous management of metabolic acidosis with bicarbonate therapy.

How well did you know this?
1
Not at all
2
3
4
5
Perfectly
69
Q

Treatment for an infant’s unilateral undescended testis should be started at what age?

A

at 6–12 months of age to avoid testicular damage.

How well did you know this?
1
Not at all
2
3
4
5
Perfectly
70
Q

Human parvovirus B19 is associated with what?

A

Parvovirus B19 is associated with erythema infectiosum (fifth disease). It is also associated with nonspecific fever, arthropathy, chronic anemia, and transient aplastic crisis.

How well did you know this?
1
Not at all
2
3
4
5
Perfectly
71
Q

Causes of stillbirth?

A

Stillbirth is defined as fetal death occurring at or after 20 weeks gestation, and Causes of stillbirth include placental abnormalities and obstetric complications, which are the two largest category. Others include infection and fetal defects.

How well did you know this?
1
Not at all
2
3
4
5
Perfectly
72
Q

Treatment for pneumonia and suspected coinfection with influenza

A

Treatment should include antiviral and antibacterial agents that include coverage against methicillin-resistant Staphylococcus aureus (MRSA), the most common bacterial pathogen isolated from critically ill patients with coinfection. Oseltamivir, ceftriaxone, azithromycin, and vancomycin should be initiated empirically.

How well did you know this?
1
Not at all
2
3
4
5
Perfectly
73
Q

Medications to start treatment for HTN in pts with CAD?

A

AHA recommends treat hypertension in patients with stable heart failure with ACE inhibitors and/or β-blockers. ACE inhibitors are recommended as antihypertensive agents in patients already on β-blocker therapy (especially following MI), in diabetics, and in pts with left ventricular dysfunction.

How well did you know this?
1
Not at all
2
3
4
5
Perfectly
74
Q

Describe symptoms of Meniere’s disease

A

Multiple episodes of vertigo lasting for 20–120 minutes, accompanied by a fluctuating hearing loss, tinnitus, and a sense of aural fullness. Audiograms will reveal a low-frequency hearing loss.

How well did you know this?
1
Not at all
2
3
4
5
Perfectly
75
Q

Monotherapy for bipolar disorder

A

Divalproex (same names Valproic Acid, Depakote) and lithium are indicated for treating bipolar depression or acute mania, and for maintenance. They should be given as single agents, not in combination with other drugs.

How well did you know this?
1
Not at all
2
3
4
5
Perfectly
76
Q

Treatment for patient with diabetes mellitus and advanced coronary artery disease

A

CABG was superior to percutaneous coronary intervention (PCI) in that it significantly reduced rates of death and myocardial infarction.

How well did you know this?
1
Not at all
2
3
4
5
Perfectly
77
Q

In young children with bilious emesis, anorexia, and lack of fever, the most likely diagnosis is:

A

Intestinal malrotation with volvulus

Diagnosis: upper GI series to evaluate initially (CT too much radiation for young children)

How well did you know this?
1
Not at all
2
3
4
5
Perfectly
78
Q

Treatment for Clostridium difficile colitis

A

Metronidazole, vancomycin, and fidaxomicin are the three medications used for Clostridium difficile colitis. Only metronidazole is effective IV. Treatment with vancomycin and fidaxomicin is PO.

How well did you know this?
1
Not at all
2
3
4
5
Perfectly
79
Q

Treatment for Calcaneal apophysitis, also called Sever’s disease

A

Calcaneal apophysitis (Sever’s disease) is a common cause of heel pain in young athletes, especially those who participate in basketball, soccer, track, and other sports that involve running. Typically the heel apophysis closes by age 15. Treatment options include activity modification, the use of ice packs and/or moist heat, stretching, analgesics, and orthotic devices.

How well did you know this?
1
Not at all
2
3
4
5
Perfectly
80
Q

Patients with persistent symptoms carpal tunnel syndrome achieve the best long-term relief with?

A

surgery

How well did you know this?
1
Not at all
2
3
4
5
Perfectly
81
Q

Treatment for SVT?

A

Vagal maneuvers, IV adenosine or verapamil, IV diltiazem or β-blockade, IV antiarrhythmics, or cardioversion in refractory cases. Radiofrequency ablation is becoming first-line therapy for all patients with recurrent SVT, not just those refractory to suppressive drug therapies.

How well did you know this?
1
Not at all
2
3
4
5
Perfectly
82
Q

Mechanical ventilation settings for Acute respiratory distress syndrome (ARDS)

A

Early recognition and prompt treatment with intubation and mechanical ventilation is necessary to improve chances for survival in ARDS. Patients with ARDS should be started at lower tidal volumes (6 mL/kg) instead of the traditional volumes (10–15 mL/kg) (SOR A). They often require higher positive end-expiratory pressure settings (PEEP) (SOR B).

How well did you know this?
1
Not at all
2
3
4
5
Perfectly
83
Q

First-line treatment for diabetic peripheral neuropathy, according to the ADA

A

TCAs. Among the TCAs, amitriptyline, imipramine, and nortriptyline have been found to be the most effective. Other options include duloxetine, pregabalin, oxycodone, and tramadol.

How well did you know this?
1
Not at all
2
3
4
5
Perfectly
84
Q

Features of fetal alcohol syndrome, Down syndrome, and Prader-Willi syndrome

A

Fetal alcohol syndrome - minor facial anomalies, including a thin upper lip, a smooth philtrum, and a flat nasal bridge. Other physical anomalies include clinodactyly; prenatal and postnatal growth retardation; and functional or structural CNS abnormalities.
Down syndrome - hypotonia, a flat face, upward and slanted palpebral fissures and epicanthic folds, and speckled irises (Brushfield spots); varying degrees of mental and growth retardation; dysplasia of the pelvis; cardiac malformations; a simian crease; short, broad hands; hypoplasia of the middle phalanx of the 5th finger; and a high, arched palate.
Prader-Willi syndrome - severe hypotonia at birth, obesity, short stature (responsive to growth hormone), small hands and feet, hypogonadism, and mental retardation.

How well did you know this?
1
Not at all
2
3
4
5
Perfectly
85
Q

Drugs that can cause drug induced SLE

A

hydralazine, antiarrhythmics

How well did you know this?
1
Not at all
2
3
4
5
Perfectly
86
Q

Defined orthostatic hypotension

A

Drop in blood pressure of at least 20 mm Hg systolic or 10 mm Hg diastolic within 3 minutes of standing.

How well did you know this?
1
Not at all
2
3
4
5
Perfectly
87
Q

Describe cluster A, B, C type of personality disorders?

A

Cluster A personality disorder group - odd or eccentric personalities. Group includes paranoid, schizoid, and schizotypal personality disorders.
Cluster B disorders - dramatic, emotional, or erratic personalities. Group includes antisocial, borderline, histrionic, and narcissistic personality disorders.
Cluster C disorders - anxious or fearful behaviors. Group includes obsessive-compulsive and dependent personality disorders.

How well did you know this?
1
Not at all
2
3
4
5
Perfectly
88
Q

Describe intertrigo, most common organism in intertrigo, and treatment

A

Intertrigo is skin inflammation caused by skin-on-skin friction. Due to moisture trapped in deep skinfolds where air circulation is limited. When intertrigo does not respond to usual conservative measures, including keeping the skin clean and dry, evaluation for infection is recommended. A Wood’s light examination, KOH preparation, and exudate culture can assist in identifying causative organisms.
The moist, damaged skin is breeding ground for various microorganisms, and secondary cutaneous infections are common. Candida is organism most commonly associated with intertrigo.

How well did you know this?
1
Not at all
2
3
4
5
Perfectly
89
Q

Treatment for acute rectal fissure

A

Although patients often require an internal sphincterotomy, nonsurgical measures that relax the sphincter have proven helpful. Botulinum toxin injected into the internal sphincter has proven most beneficial, but topical preparations are not yet available and have not been shown to be effective. Drugs that dilate the internal sphincter, including diltiazem, nifedipine, and nitroglycerin ointment, have proven to be beneficial in healing acute fissures

How well did you know this?
1
Not at all
2
3
4
5
Perfectly
90
Q

Describe signs of the four levels of alcohol withdrawal severity: minor, major, seizures, and delirium tremens

A

Chronic excessive alcohol intake produces functional changes in neurotransmitter activity that can lead to a net increase in excitatory neuroreceptor activity when the person stops drinking. Withdrawal can be divided into four levels of severity: minor, major, seizures, and delirium tremens.

  • Minor alcohol withdrawal is characterized by tremor, anxiety, nausea, vomiting, and/or insomnia 6–24 hours after the patient’s last drink.
  • Major withdrawal occurs 10–72 hours after the last drink and can include the signs and symptoms of minor withdrawal, as well as visual and auditory hallucinations, diaphoresis, tachycardia, and elevated blood pressure.
  • Alcoholic seizure generally occurs within 2 days of the last drink and may be the only sign of withdrawal, although approximately one-third of these patients will progress to delirium tremens.
  • Delirium tremens can occur anytime within 3–10 days following the last drink. The defining clinical finding is delirium, but the findings seen in milder forms of alcohol withdrawal can also be present, and may be more severe. Fever is most often seen with delirium tremens and is less common with less severe forms of alcohol withdrawal.
How well did you know this?
1
Not at all
2
3
4
5
Perfectly
91
Q

Additional heart failure treatment to add when standard treatment is insufficient to control symptoms or cannot be tolerated.

A

Combination of the vasodilators hydralazine and isosorbide dinitrate has been shown to be effective in the treatment of heart failure when standard treatment with diuretics, β-blockers, and an ACE inhibitor (or ARB) is insufficient to control symptoms or cannot be tolerated. This combination is particularly effective in African-Americans with NYHA class III or IV heart failure, with advantages of reduced mortality rates and improvement in quality-of-life measures.

How well did you know this?
1
Not at all
2
3
4
5
Perfectly
92
Q

Signs of heroin overdoses

A

Heroin overdose is manifested by CNS depression and hypoventilation. Clues include pupillary miosis and a decreasing respiratory rate in the presence of a semi-wakeful state. In addition to hypoventilation, a multifactorial acute lung injury occurs within 2–4 hours of the overdose and is associated with hypoxemia and a hypersensitivity reaction, resulting in noncardiogenic pulmonary edema. Findings include hypoxia, crackles on lung auscultation, and pink, frothy sputum. Treatment must include respiratory support with intubation, mechanical ventilation, oxygen and opiate reversal with naloxone, which may require repeat doses or intravenous infusion.

How well did you know this?
1
Not at all
2
3
4
5
Perfectly
93
Q

First choice for chronic potent opioid therapy for chronic pain

A

Morphine is the best first choice for chronic potent opioid therapy (SOR B). It is reliable and inexpensive, and equivalent doses can be easily calculated if the patient must later be switched to another medication. Transdermal fentanyl and hydromorphone are reasonable second-line choices. They are not recommended as first-line therapy because they are expensive and can produce tolerance relatively quickly

How well did you know this?
1
Not at all
2
3
4
5
Perfectly
94
Q

FDA approved antibiotic for acne rosacea

A

Doxycycline at a subantimicrobial dosage (40 mg daily). This does not contribute to antibiotic resistance, even when used over several months, and is better tolerated than higher dosages.

How well did you know this?
1
Not at all
2
3
4
5
Perfectly
95
Q

You suspect that he has asthma. What do you do first?

A

Do spirometry. The American Academy of Asthma, Allergy, and Immunology recommends that asthma not be diagnosed or treated without spirometry first.

How well did you know this?
1
Not at all
2
3
4
5
Perfectly
96
Q

How to differentiate abdominal pain due to visceral vs within the abdominal wall itself

A

Palpate the tender location of abdomen with patient in supine position. Then have patient raise both legs off table while you palpate abdomen. A reduction of the pain caused by abdominal palpation when the abdominal muscles are tightened is known as Carnett’s sign. If the cause of the pain is visceral, the taut abdominal muscles may protect the locus of pain. Intensification of pain with this maneuver points to a source of pain within the abdominal wall itself.

How well did you know this?
1
Not at all
2
3
4
5
Perfectly
97
Q

Mild cognitive impairment is characterized by what

A

Mild cognitive impairment is an intermediate stage between normal cognitive function and dementia. Motor function remains normal. Patients have normal functional activities but there is objective evidence of memory impairment.

How well did you know this?
1
Not at all
2
3
4
5
Perfectly
98
Q

Describe Centor criteria

A

Centor criteria are the clinical predictors of streptococcal pharyngitis. Include tonsillar exudates, tender anterior cervical lymphadenopathy, absence of cough, and history of fever.

Patients with four positive criteria should be treated with antibiotics, those with three positive criteria should be tested and treated if positive, and those with 0–1 positive criteria should be treated with analgesics and supportive care only.

How well did you know this?
1
Not at all
2
3
4
5
Perfectly
99
Q

Routine vaccination against which one of the following organisms has significantly reduced the risk of bacterial meningitis among young children?

A

Conjugate vaccines against Haemophilus influenzae type b and Streptococcus pneumoniae have been highly effective in reducing the incidence of bacterial meningitis in young children.

How well did you know this?
1
Not at all
2
3
4
5
Perfectly
100
Q

Describe long QT syndrome

A

Patients with EKGs showing a QTc interval >480 ms with a syncopal episode, or >500 ms in the absence of symptoms, are diagnosed with long QT syndrome if no secondary cause such as medication use is present. Consists of cardiac repolarization defects. Associated with polymorphic ventricular tachycardia, including torsades de pointes, and sudden cardiac death. Treat with p-blockers and implanted cardioverter defibrillators.

How well did you know this?
1
Not at all
2
3
4
5
Perfectly
101
Q

When should an antibiotic time-out be scheduled at the time a patient is admitted to the hospital?

A

For patients started on empiric antibiotic therapy at hospital admission, the CDC recommends an antibiotic time-out 48 hours after the initial order to determine if it can be stopped or needs to be changed.

How well did you know this?
1
Not at all
2
3
4
5
Perfectly
102
Q

Treatments that can improve symptoms of Irritable bowel syndrome (IBS) symptoms.

A

Irritable bowel syndrome (IBS) symptoms improve with several different medications and alternative therapies. Exercise, probiotics, antibiotics, antispasmodics, antidepressants, psychological treatments, and peppermint oil all have evidence that they may improve IBS symptoms (SOR B). Antidepressants can improve pain and overall symptom scores compared to placebo. SSRIs used in these trials included citalopram, fluoxetine, and paroxetine, and tricyclic antidepressants included amitriptyline, desipramine, and imipramine.

How well did you know this?
1
Not at all
2
3
4
5
Perfectly
103
Q

Backup contraception after IUD insertion.

A

Patient does not need to use backup contraception if IUD is inserted within 7 days after menstrual bleeding started. If the levonorgestrel IUD is inserted more than 7 days after menstrual bleeding starts, then patient needs to abstain from sexual intercourse or use additional contraceptive protection for the next
7 days.

How well did you know this?
1
Not at all
2
3
4
5
Perfectly
104
Q

PFT for restrictive lung disease vs obstructive lung disease.

A

A reduced FVC with either a normal or increased FEV1/FVC ratio is consistent with restrictive lung disease. There are three basic categories of restrictive lung disease: intrinsic lung disease, chest wall deformities, and neuromuscular disorders. A reduced FEV1 and decreased FEV1/FVC ratio is seen in obstructive lung disease (asthma, COPD). The DLCO is the measure of the diffusion of carbon monoxide across the alveolar-capillary membrane. Reduced values are obtained when interstitial fibrosis is extensive, or when the capillary surface is compromised by vascular obstruction or nonperfusion, or is destroyed (as in emphysema). A bronchodilator challenge will allow assessment of reversible airway obstruction. A methacholine challenge test looks for airway hyperreactivity.

How well did you know this?
1
Not at all
2
3
4
5
Perfectly
105
Q

Two causes of pain at the first metatarsophalangeal (MTP) joint of foot.
Sign of Morton’s neuroma.

A

The first metatarsophalangeal (MTP) joint has two sesamoid bones that can be injured. Overuse, a sharp blow, and sudden dorsiflexion are the most common mechanisms of injury.

Gout often involves the first MTP joint, but the onset is sudden, with warmth, redness, and swelling, and pain on movement of the joint is common.

Morton’s neuroma causes numbness involving the digital nerve in the area, and usually is caused by the nerve being pinched between metatarsal heads in the center of the foot.

How well did you know this?
1
Not at all
2
3
4
5
Perfectly
106
Q

Treatment for Raynaud’s disease

A

No currently approved treatment for Raynaud’s disease. Patients reportedly experience subjective symptomatic improvement with dihydropyridine calcium channel antagonists, with nifedipine being the calcium channel blocker of choice. (1-Antagonists such as prazosin or terazosin are also effective. B-Blockers can produce arterial insufficiency of the Raynaud type, so propranolol and atenolol are contraindicated.

How well did you know this?
1
Not at all
2
3
4
5
Perfectly
107
Q

Treatment for acute epididymo-orchitis

A

Acute epididymitis is often the result of descending infection caused by urinary tract pathogens. When the infection involves the epididymis and testis (epididymo-orchitis), sonography will frequently show an enlarged heterogeneous testis with increased color flow. Give antibiotics for the treatment of both gonorrhea and Chlamydia infections.

How well did you know this?
1
Not at all
2
3
4
5
Perfectly
108
Q

The five Ps of acute compartment syndrome

A

Typically, compartment pressure can be measured using a needle attached to a manometer, and if the pressure is elevated (usually >40 mm Hg) urgent fasciotomy is necessary to prevent muscle necrosis. If the classic “Five Ps” (pain, paresthesia, pallor, pulselessness, and paralysis) are all present, the outcome will most certainly be bad, even limb-threatening.

Before the classic findings develop, patients will have tenderness out of proportion to the physical appearance of the injury and, most importantly, severe pain in the involved compartment with passive stretching of the involved muscles.

How well did you know this?
1
Not at all
2
3
4
5
Perfectly
109
Q

Associated conditions with Crohn’s disease

A

Crohn’s disease is associated with many extragastrointestinal conditions: erythema nodosum (as with this patient), anemia, inflammatory arthropathies, uveitis, and venous thromboembolism (level of evidence 3).

How well did you know this?
1
Not at all
2
3
4
5
Perfectly
110
Q

Adverse effect of being on long term opioid therapy

A

Hypogonadism is often underrecognized and undertreated side effect of long-term opioid therapy. More often seen in men and in patients receiving larger doses of opioids. Typical symptoms include decreased libido, erectile dysfunction, amenorrhea, or fatigue. Also constipation

How well did you know this?
1
Not at all
2
3
4
5
Perfectly
111
Q

Treatment for mild to moderate croup and moderate to severe croup

A

Croup is a viral infection that results in swelling in the larynx. It rarely is severe enough to cause respiratory collapse or require intubation and must be differentiated from more severe conditions such as epiglottitis, retropharyngeal abscess, or pneumonia. Condition is usually benign and self-limiting, with the worst symptoms occurring at night. Treat mild to moderate croup with a single dose of either an oral or IM corticosteroid. Dexamethasone is recommended due to its 72-hour length of effect. Inhaled racemic epinephrine has been shown to reduce the need for intubation in cases of moderate to severe croup.

How well did you know this?
1
Not at all
2
3
4
5
Perfectly
112
Q

fungal disease regions in US

A

Histoplasmosis - mid US (Iowa, Nebraska, etc)
Coccidioidomycosis - southwest US (California, AZ, NM)
Blastomycosis - east US

How well did you know this?
1
Not at all
2
3
4
5
Perfectly
113
Q

Describe lichen planus

A

Lichen planus is an idiopathic inflammatory disease affecting the skin and oral mucosa. The characteristic violaceous, polygonal papules on skin may be intensely itchy. Can cause oral ulcers. There is a significant association between lichen planus and hepatitis C virus infection.

How well did you know this?
1
Not at all
2
3
4
5
Perfectly
114
Q

Describe likelihood ratio (LR)

A

The likelihood ratio (LR) is the ratio of the probability of a specific test result in people who have a particular disease to the probability in people who do not. LRs correspond to how well a test rules in or rules out a given disease. A test with an LR of 1.0 indicates that it does not change the probability of disease. The higher above 1 the LR is, the more likely it is that the disease is present (an LR >10 is considered good). Conversely, the lower the LR is below 1, the more likely it is that the disease is not present (an LR <0.1 is considered good).

How well did you know this?
1
Not at all
2
3
4
5
Perfectly
115
Q

Adverse effects of bisphosphonates

A

The use of bisphosphonates is associated with a small increase in the risk of atypical femoral shaft fractures. The risk increases with the duration of use (SOR B). Also associated with increased risk of osteonecrosis of the jaw, esophagitis, esophageal ulceration, and hypocalcemia. Bisphosphonates are used as a treatment for hypercalcemia.

How well did you know this?
1
Not at all
2
3
4
5
Perfectly
116
Q

What to do if you suspect developmental dysplasia of the hip

A

Developmental dysplasia of the hip encompasses both subluxation and dislocation of the newborn hip, as well as anatomic abnormalities. It is more common in firstborns, females, breech presentations, oligohydramnios, and patients with a family history of developmental dysplasia.

If there is any question of a hip problem on examination by 2 weeks of age, the recommendation is to refer to a specialist for further testing and treatment. Studies show that these problems disappear by 1 week of age in 60% of cases, and by 2 months of age in 90% of cases.

Closed reduction and immobilization in a Pavlik harness, with ultrasonography of the hip to ensure proper positioning, is the treatment of choice until 6 months of age. The American Academy of Pediatrics recommends ultrasound screening at 6 weeks for breech females, breech males (optional), and females with a positive family history of developmental dysplasia of the hip.

How well did you know this?
1
Not at all
2
3
4
5
Perfectly
117
Q

Choice of medication for chronic pain in ESRD

A

Fentanyl is metabolized in the liver and has no active metabolites. Codeine, hydrocodone, morphine all have active metabolites that can accumulate in patients with renal failure, leading to serious side effects. Avoid in patients on dialysis.

How well did you know this?
1
Not at all
2
3
4
5
Perfectly
118
Q

Pattern of low diffusing capacity of the lung for carbon monoxide (DLCO) and PFT in conditions below:
Chronic pulmonary embolism
Emphysema
Interstitial lung disease and hypersensitivity pneumonitis
Asthma

A

A diffusion capacity test assesses how well a tracer gas in inspired air can cross from the air into the blood. The diffusion capacity provides a general assessment of the air-blood interface. Reduced values are seen with severe interstitial fibrosis, or when the capillary surface has been compromised by vascular obstruction (pulmonary embolism) or is destroyed by emphysema.

Chronic pulmonary embolism - low diffusing capacity of the lung for carbon monoxide (DLCO) and normal PFT
Emphysema - low DLCO and an obstructive PFT
Interstitial lung disease and hypersensitivity pneumonitis - low DLCO and restrictive PFTs.
Asthma - may have an increased DLCO with an obstructive PFT, with reversibility after bronchodilator administration.

How well did you know this?
1
Not at all
2
3
4
5
Perfectly
119
Q

Muscle strength score

A

Muscle strength is scored on a scale of 0 to 5.

0 - The inability to contract a muscle.
1 - Contraction without movement.
2 - Movement with the effect of gravity neutralized
3 - Movement against gravity only
4 - Movement against gravity plus some resistance.
5 - Normal, Movement against substantial resistance.

How well did you know this?
1
Not at all
2
3
4
5
Perfectly
120
Q

USPSTF recommendation annual screening for lung cancer with low-dose computed tomography (LDCT)

A

The U.S. Preventive Services Task Force recommends annual screening for lung cancer with low-dose computed tomography (LDCT) in adults 55–80 years of age who have a 30-pack-year smoking history and currently smoke or have quit within the past 15 years. Screening should be discontinued once a person has not smoked for 15 years or develops a health problem that substantially limits life expectancy or the ability or willingness to have lung surgery (B recommendation).

How well did you know this?
1
Not at all
2
3
4
5
Perfectly
121
Q

cluster headaches treatment - prevention and abortive tx

A

Verapamil is first line prophylactic agent for the prevention of cluster headaches (SOR A). Oxygen and sumatriptan are first-line abortive therapies for cluster headaches (SOR A).

How well did you know this?
1
Not at all
2
3
4
5
Perfectly
122
Q

What conditions decreases or increases INR when on coumadin?

A

Medical conditions that decrease responsiveness to warfarin and reduce the INR include hypothyroidism, visceral carcinoma, increased vitamin K intake, diabetes mellitus, and hyperlipidemia. Conditions that increase responsiveness to warfarin, the INR, and the risk of bleeding include vitamin K deficiency caused by decreased dietary intake, malabsorption, scurvy, malnutrition, cachexia, small body size, hepatic dysfunction, moderate to severe renal impairment, hypermetabolic states, fever, hyperthyroidism, infectious disease, heart failure, and biliary obstruction (SOR B, SOR C).

How well did you know this?
1
Not at all
2
3
4
5
Perfectly
123
Q

Which class of oral diabetes meds are the most likely to cause hypoglycemia?

A

Sulfonylureas are the oral hypoglycemic agents most likely to cause hypoglycemia, with glyburide more likely than glipizide due to its longer half-life. The use of these agents should be rare in elderly patients with diabetes mellitus.

How well did you know this?
1
Not at all
2
3
4
5
Perfectly
124
Q

If patient received BCG vaccine, what test for TB is best?

A

Immigrants who received the bacille Calmette-Guérin (BCG) vaccine can have false-positive tests. Interferon-y release assays (QuantiFERON-TB Gold Plus) preferred to tuberculin skin testing in immigrants who have been vaccinated with BCG.

How well did you know this?
1
Not at all
2
3
4
5
Perfectly
125
Q

which has the highest specificity for heart failure

A

Among the constellation of history and physical findings that can be found in patients with heart failure, none provides a proof-positive diagnosis alone, as most are found in other disease states as well. Each of the options listed raises the possibility of heart failure but the only one that has a specificity >90% is the third heart sound, which is 99% specific for the diagnosis of heart failure. Other findings with >90% sensitivity include a displaced point of maximal impulse, interstitial edema or venous congestion on a chest radiograph, jugular vein distention, and hepatojugular reflux. The other options listed here have specificities for heart failure that fall within the range of 65%–80%.

How well did you know this?
1
Not at all
2
3
4
5
Perfectly
126
Q

Which cancer is mostly likely seen with Courvoisier’s sign?

A

The presence of a solitary enlarged left supraclavicular lymph node (Virchow’s node) is associated with a gastrointestinal system malignancy. When combined with painless jaundice and a palpable nontender gallbladder (Courvoisier’s sign), pancreatic cancer is the most likely diagnosis.

How well did you know this?
1
Not at all
2
3
4
5
Perfectly
127
Q

What conditions are beneficial with hyperbaric oxygen?

A

Medical hyperbaric oxygen is considered a reimbursable treatment option by many insurers for a long list of diagnoses. The list of conditions shown to benefit from hyperbaric oxygen is a shorter one, however, and includes decompression sickness and wounds caused by crush injuries.

How well did you know this?
1
Not at all
2
3
4
5
Perfectly
128
Q

Duloxetine is what class of antidepressant and what can it treat

A

Duloxetine (Cymbalta) is SNRI. Treats depression, chronic pain, does not cause weight gain

How well did you know this?
1
Not at all
2
3
4
5
Perfectly
129
Q

Next step in microscopic hematuria finding

A

Initially assess for benign causes such as urinary tract infection, vigorous exercise, menstruation, and recent urologic procedures. If none of these is found, next step would be assessing for renal disease using urine microscopy to look for casts or dysmorphic blood cells, and checking renal function. If the results are negative, CT urography and cystoscopy should be performed. CT evaluates the upper urinary tract for nephrolithiasis and renal cancer, while cystoscopy evaluates the bladder for bladder cancer, urethral strictures, and prostatic problems.

How well did you know this?
1
Not at all
2
3
4
5
Perfectly
130
Q

What are differentials for with pink, scaling papules and plaques on the trunk and proximal aspect of the arms and legs in young adult.

A

Differential diagnosis of multiple small scaling plaques includes drug eruptions, secondary syphilis, guttate psoriasis, and erythema migrans.

How well did you know this?
1
Not at all
2
3
4
5
Perfectly
131
Q

Most common extra-articular manifestation of ankylosing spondylitis (AS) and treatment for it

A

Uveitis is inflammation of the uveal tract and can affect any or all of its components, including the iris. It is the most common extra-articular manifestation of ankylosing spondylitis (AS). Iritis presents with a painful red eye with conjunctival injection, photophobia, and a sluggishly reacting pupil. A hazy-appearing anterior chamber results from the iris producing an inflammatory exudate. Treatment is topical corticosteroids but oral or parenteral corticosteroids and NSAIDs are also effective. Reduced anterior spine flexion results from the skeletal manifestations of AS. A “bamboo spine” is classically seen on lumbar radiographs

How well did you know this?
1
Not at all
2
3
4
5
Perfectly
132
Q

Imagining to order for elevated prolactin level

A

In almost all patients with an elevated prolactin level, MRI of the pituitary is recommended to exclude the possibility of a pituitary adenoma (SOR C).

How well did you know this?
1
Not at all
2
3
4
5
Perfectly
133
Q

Alternative antibiotic to penicillin or amoxicillin allergy

A

Risk of a serious allergic reaction to second-and third-generation cephalosporins in patients with penicillin or amoxicillin allergy appears to be almost nil, and no greater than the risk among patients without such allergies.

How well did you know this?
1
Not at all
2
3
4
5
Perfectly
134
Q

Which EKG abnormalities would dictate the use of a pharmacologic stress test as opposed to an exercise stress test?

A

Left bundle branch block makes the EKG uninterpretable during an exercise stress test, and can also interfere with nuclear imaging performed during the test. It is associated with transient positive defects in the anteroseptal and septal regions in the absence of a lesion within the left anterior descending coronary artery. This leads to a high rate of false-positive tests and low specificity. Pharmacologic stress tests using vasodilators such as adenosine with nuclear imaging have a much higher specificity and positive predictive value for LAD lesions, and the same is true for dobutamine stress echocardiography, which is why these are the preferred methods for evaluating patients with left bundle branch block.

How well did you know this?
1
Not at all
2
3
4
5
Perfectly
135
Q

Treatment for poison ivy dermatitis

A

Poison ivy dermatitis is caused by urushiol, a resin found in poison ivy, poison oak, and poison sumac plants. This is a type IV T cell–mediated allergic reaction, so it typically takes at least 12 hours and often 2–3 days before the reaction is fully manifested. Depending on the degree of contact (i.e., the amount of resin on the skin), the rash often progresses over a couple of days, giving the impression that it is spreading. Also, delayed contact with resin from contaminated clothing, gloves, or pets may result in new lesions appearing over several days. Brushing against the leaves of the plant causes the linear streaking pattern characteristic of poison ivy dermatitis. The resin can be inactivated with any type of soap, thereby preventing the reaction, but the sooner the better. Approximately 50% of the resin can be removed by soap and water within 10 minutes of contact, but after 30 minutes only about 10% can still be removed. Treat with antihistamines for the itching and expert consensus is to use a higher dosage tapered over a longer period, generally 10–14 days, in order to prevent a relapse (such as oral prednisone)

How well did you know this?
1
Not at all
2
3
4
5
Perfectly
136
Q

What is important to exclude before undergoing anesthesia for surgery in a patient with long-standing rheumatoid arthritis

A

Imaging of the patient’s cervical spine to detect atlantoaxial subluxation in a RA would be most important for preventing a catastrophic spinal cord injury during intubation. In many cases cervical fusion must be performed before other elective procedures can be contemplated.

How well did you know this?
1
Not at all
2
3
4
5
Perfectly
137
Q

In HIV patients with immunodeficiency, what antibiotics should be given for prophylactic treatment based on CD4 counts?

A

Patients with HIV infection and severe immunodeficiency are at risk for certain opportunistic infections. Susceptibility to opportunistic infections can be measured by CD4+ T lymphocyte counts. Patients with a CD4+ count <200 cells/mm3 should receive trimethoprim/sulfamethoxazole for prophylaxis against Pneumocystis pneumonia, and prophylaxis against Toxoplasma gondii when CD4+ level is <100 cells/mm3. Azithromycin for prophylaxis against Mycobacterium avium-intracellulare complex when CD4+ counts are <50 cells/mm3. Itraconazole for prophylaxis against Histoplasma capsulatum infection when the CD4+ count is :150 cells/mm3 if the patient is at risk due to occupational exposure or living in a community with a hyperendemic rate of histoplasmosis (>10 cases per 100 patient years).

How well did you know this?
1
Not at all
2
3
4
5
Perfectly
138
Q

Describe Legg-Calvé-Perthes disease

A

Legg-Calvé-Perthes disease results from interruption of the blood supply to the still-growing femoral head. It occurs in children 2–12 years of age and presents with hip pain and an atraumatic limp. Common physical findings include leg-length discrepancies, and limited abduction and internal rotation. Radiographs reveal sclerosis of the proximal femur with joint space widening. MRI confirms osteonecrosis.

How well did you know this?
1
Not at all
2
3
4
5
Perfectly
139
Q

First-line therapy for menstrual abnormalities, hirsutism, and acne in polycystic ovary syndrome

A

Hormonal contraceptives are the first-line therapy for menstrual abnormalities, hirsutism, and acne in polycystic ovary syndrome. Clomiphene is used for infertility. Metformin is beneficial for metabolic/glycemic abnormalities and menstrual irregularities, but does not improve hirsutism or acne.

How well did you know this?
1
Not at all
2
3
4
5
Perfectly
140
Q

What is recommendation for feeding for older adults with advanced dementia?

A

The American Geriatrics Society (AGS) states that percutaneous feeding tubes are not recommended for older adults with advanced dementia, and that careful hand feeding should be offered instead. This is the first recommendation by the AGS in the Choosing Wisely campaign. Careful hand feeding for patients with severe dementia is at least as good as tube feeding with regard to the outcomes of death, aspiration pneumonia, functional status, and patient comfort. Regular food is preferred. Tube feeding is associated with agitation, increased use of physical and chemical restraints, and worsening pressure ulcers.

How well did you know this?
1
Not at all
2
3
4
5
Perfectly
141
Q

What do people on the National Weight Control Registry do?

A

Individuals on the National Weight Control Registry eat a low-fat diet rich in complex carbohydrates, eat breakfast daily, weigh themselves at least once a week, and are physically active for 60–90 minutes a day.

How well did you know this?
1
Not at all
2
3
4
5
Perfectly
142
Q

What makes type 1 DM more likely thank type 2 DM

A

The patient presents with marked hyperglycemia but no evidence of ketoacidosis or nonketotic coma. Differentiating between type 1 and type 2 diabetes mellitus is important for guiding therapy. The gradual onset of symptoms is more consistent with type 2 diabetes mellitus, whereas type 1 diabetes typically has a more rapid onset. Patients with type 1 diabetes typically need lower doses of insulin to correct hyperglycemia, as they lack the insulin insensitivity that is the hallmark of type 2 diabetes. Positive anti-GAD antibodies and low C-peptide at the time of the initial diagnosis are also consistent with type 1 diabetes, although C-peptide levels can also be low in long-standing type 2 diabetes. Weight loss occurs in both types of diabetes mellitus when glucose is profoundly elevated.

How well did you know this?
1
Not at all
2
3
4
5
Perfectly
143
Q

Atypical antipsychotics is most appropriate for which condition

A

Atypical antipsychotics (2nd generation antipsychotics) may reduce the duration of delirium in adult intensive-care patients, and are recommended by the American College of Critical Care Medicine in their clinical practice guidelines for the management of pain, agitation, and delirium in adult patients in the intensive-care unit.

The American Geriatrics Society and the American Psychiatric Association (APA) recommend not using antipsychotics as a first choice to treat the behavioral and psychological symptoms associated with dementia, such as aggression and resistance to care. These drugs have limited benefit and can cause serious harm, including stroke and premature death.

Typical antipsychotic is haloperidol (first generation antipsychotic)

How well did you know this?
1
Not at all
2
3
4
5
Perfectly
144
Q

Describe polymyositis/dermatomyositis group

A

Polymyositis/dermatomyositis group is an inflammatory myopathy. Proximal muscle involvement and elevation of serum muscle enzymes such as creatine kinase and aldolase are characteristic. Corticosteroids are the accepted treatment of choice.

How well did you know this?
1
Not at all
2
3
4
5
Perfectly
145
Q

Which one of uterotonic agents is CONTRAINDICATED in the management of postpartum hemorrhage if patient has hypertension or asthma?
Methylergonovine, Oxytocin (Pitocin), misoprostol (Cytotec), Carboprost tromethamine (Hemabate)

A

Uterotonics are the first-line treatment for postpartum hemorrhage in patients with decreased uterine tone. Methylergonovine should be avoided if the patient is hypertensive. It is an ergot alkaloid that causes generalized smooth muscle contraction and can raise blood pressure. Oxytocin (Pitocin) and misoprostol (Cytotec) not have any contraindications. Carboprost tromethamine (Hemabate) be avoided in asthmatic patients and is relatively contraindicated if the patient has hepatic, renal, or cardiac disease.

How well did you know this?
1
Not at all
2
3
4
5
Perfectly
146
Q

In a healthy premenopausal women, treatment for UTI is based on what?

A

In healthy premenopausal women with no history of a urinary tract functional abnormality, current pregnancy, or another underlying condition that may increase the risk of treatment failure, infection can be presumed based on symptoms. Patients with acute uncomplicated cystitis are not febrile and have no vaginal symptoms. Treat based on the symptoms rather than documented evidence of infection (SOR C).

How well did you know this?
1
Not at all
2
3
4
5
Perfectly
147
Q

Treatment for moderate osteoarthritis pain in patient with CKD3?

A

Acetaminophen is the analgesic of choice for short-term treatment of mild to moderate pain in patients with stage 3–5 chronic kidney disease. Chronic nonterminal pain requires initial treatment with nonopioid analgesics. NSAIDs should be avoided because of the risk of nephrotoxicity.

How well did you know this?
1
Not at all
2
3
4
5
Perfectly
148
Q

Blood gas for hyperventilation

A

Anxiety, shortness of breath, paresthesia, and carpopedal spasm are characteristic of hyperventilation. Respiratory alkalosis secondary to hyperventilation characterized by elevated arterial pH and depressed pCO2.

How well did you know this?
1
Not at all
2
3
4
5
Perfectly
149
Q

Distinguish starvation from cachexia in elderly patients?

A

In the frail elderly, it may be difficult to distinguish relative starvation due to decreased or inadequate caloric intake from cachexia, which is due to an inflammatory response with elevated cytokines. Appetite is decreased early in cachexia but remains normal in the early stages of starvation. Albumin decreases early in cases of cachexia and later in starvation. Due to the inflammatory changes, cachexia is resistant to refeeding. A reversal of changes occurs with refeeding in starvation

How well did you know this?
1
Not at all
2
3
4
5
Perfectly
150
Q

Which NSAID is safest for patients with a previous history of myocardial infarction?

A

All oral NSAIDs increase the risk of myocardial infarction (relative risk versus placebo from 1.5 for ibuprofen to 1.7 for celecoxib), with the exception of naproxen. Cardiac risks are greater in older patients, those with a history of cardiac events, and with higher dosages.

How well did you know this?
1
Not at all
2
3
4
5
Perfectly
151
Q

Treatment for laryngitis in children less than 3 weeks.

A

supportive care

How well did you know this?
1
Not at all
2
3
4
5
Perfectly
152
Q

What are signs of retinal vein occlusion?

A

The signs of retinal vein occlusion include sudden painless loss of vision or distortion of vision. Tortuous and dilated retinal veins on funduscopic examination. Patients also often have multiple cotton-wool spots, although these are not specific to retinal vein occlusion. An afferent pupillary defect often occurs on the affected side. Diabetes mellitus and hypertension are both risk factors for retinal vein occlusion. Redness is not typical and should cause the clinician to suspect an alternate diagnosis.

How well did you know this?
1
Not at all
2
3
4
5
Perfectly
153
Q

Which drugs can cause a false-positive test for opioids on UDS?

A

Dextromethorphan, diphenhydramine, ibuprofen, and even fluoroquinolones are among the many agents that can cause a false-positive urine drug screen for opioids. Pseudoephedrine can cause a false-positive test for amphetamines

How well did you know this?
1
Not at all
2
3
4
5
Perfectly
154
Q

Stretching has NO demonstrable benefit for which condition?

A

No benefit: Joint contracture

Benefits: Hamstring strain, Chronic neck pain, Osteoarthritis, Rehabilitation post knee replacement

How well did you know this?
1
Not at all
2
3
4
5
Perfectly
155
Q

Which has been shown to be LEAST effective in the treatment of irritable bowel syndrome?

A

Fiber is ineffective in the treatment of adult irritable bowel syndrome (IBS) (SOR A).

Symptoms do improve with exercise, probiotics, antibiotics, antispasmodics, antidepressants, psychological treatments, and peppermint oil (SOR B). SSRIs used in these trials included citalopram, fluoxetine, and paroxetine, and TCAs included amitriptyline, desipramine, and imipramine.

How well did you know this?
1
Not at all
2
3
4
5
Perfectly
156
Q

Adverse effects of stimulant medications for adults with ADHD?

A

Stimulants are preferred over nonstimulant medications for adults with attention-deficit disorder. Stimulants can aggravate psychosis, tics, or hypertension and are contraindicated in patients with these problems. Main side effects of these drugs include insomnia, dry mouth, weight loss, headaches, and anxiety.

How well did you know this?
1
Not at all
2
3
4
5
Perfectly
157
Q

Antihypertensive treatment should not be started in the first 24 hours after an acute stroke unless blood pressure exceeds what?

A

AntihypertensiveElevated blood pressure may have a protective effect in the initial period after an ischemic stroke, and studies have shown adverse outcomes when it is lowered in the acute period. Blood pressure usually will spontaneously decrease without treatment in the first several hours after presentation, and antihypertensive treatment should not be started in the first 24 hours after an acute stroke unless blood pressure exceeds 220/120 mm Hg, or treatment is warranted because of another medical condition such as acute myocardial infarction. Tighter blood pressure control becomes more important after the first 24 hours.

158
Q

Site of incision for emergency tracheotomy

A

Best site for the incision in emergency tracheotomy is directly above the cricoid cartilage, through the cricothyroid membrane.

159
Q

Which antihypertensive medication been shown to slow cortical bone loss in postmenopausal females and to reduce the incidence of osteoporosis and hip fractures in those who take it continuously?

A

Thiazide diuretics have also been shown to slow cortical bone loss in postmenopausal females and to reduce the incidence of osteoporosis and hip fractures in those who take it continuously. This protective beneficial side effect disappears within 4 months following discontinuation. Disadvantages of thiazide use include excessive urinary losses of potassium and sodium and possible increases in serum glucose levels.

160
Q

What is next step if you suspect either transient synovitis or septic arthritis of the hip in a child?

A

The presentation is typical of either transient synovitis or septic arthritis of the hip. Because the conditions have very different treatment regimens and outcomes, it is important to differentiate the two. It is recommended that after plain films, the first studies to be performed should be a CBC and an erythrocyte sedimentation rate (ESR).

161
Q

Recommended initial imaging for acute pancreatitis?

A

The American College of Gastroenterology recommends transabdominal ultrasonography for all patients with acute pancreatitis (strong recommendation, low quality evidence). Contrast-enhanced CT and MRI should be reserved for patients who have an unclear diagnosis, are not clinically improving after 48–72 hours, or develop complications.

162
Q

Which is best used for calcium supplement in adults?

A

Women over the age of 50 require at least 1200 mg of calcium a day. Inadequate calcium intake is common, particularly in older women, and is associated with increased bone loss and an increased fracture risk. Calcium citrate is less dependent on stomach acidity for absorption and it may be used with long-term gastric acid suppression agents. It may be taken without regard to food or meals.

163
Q

Next step if you suspect spinal epidural abscess?

A

MRI, then IV antibiotics along with surgical debridement, if the MRI demonstrates the underlying problem.

164
Q

Do you treat subclinical hypothyroidism?

A

No, bc no proven benefit. Sx resolved within 5 years without intervention in 60% of cases. Repeat TSH with Free T4 in 1-3 months.

165
Q

Treatment for lumbar spinal stenosis

A

No strong evidence for effective nonsurgical approaches and high rates of complications with surgery. PT best evidence for initial approaches.

166
Q

Most common complication of sleeve gastrectomy

A

GERD. Sleeve gastrectomy does no produce malabsorption component so cholelithiasis, dumping syndrome, and SBO are not as likely as with other bariatric procedures.

167
Q

hypertensive urgency

A

BP >180/110 without end organ damage. Treat outpatient. If symptomatic (HA, epistaxis, etc), can use clonidine, labetalol, or captopril to lower BP.

168
Q

Treatment for seasonal affective disorder (SAD)

A

Bupropion is beneficial for prevention of SAD. Exercise and CBT are adjuncts but do not prevent SAD.

169
Q

USPSTF guideline for lung cancer screening in smokers

A

Annual low dose CT in patients 50-80 who have a 20 pack year and currently smoke or have smoked within last 15 years.

170
Q
Describe these musculoskeletal conditions.
Peroneal tendinopathy
Calcaneal apophysitis (Sever's disease)
Calcaneal stress fracture
Plantar fasciitis
Tarsal tunnel syndrome
A
Peroneal tendinopathy -  degeneration of the peroneal tendon that involves pain or tenderness in the lateral calcaneus below the ankle along the path to the base of the fifth metatarsal. 
Calcaneal apophysitis (Sever's disease) - Growth-related injury affects adolescents between 8 and 12 years of age. Symptoms often present after a growth spurt or starting a new high-impact sport or activity, and common exam findings of tight heel cords and a positive calcaneal squeeze test.
Calcaneal stress fracture - involves pain that intensifies with activity and often worsens to include pain at rest
Plantar fasciitis - sharp, shooting pain in the arch and medial aspect of the foot that often is worse upon arising and taking the first few steps of the morning
Tarsal tunnel syndrome - Entrapment of the posterior tibial nerve and causes nerve pain and numbness that radiates into the plantar aspect of the foot, often into the toes. Pain associated with tarsal tunnel syndrome typically worsens with activity and is relieved with rest.
171
Q

Which diabetes medication is not associated with hypoglycemia?

A

GLP-1 receptor agonist

172
Q

According to the Choosing Wisely campaign, adding to a threshold of how many medications should prompt thorough review to determine if any can be stopped.

A

5

173
Q

Probiotics may reduce what in patients taking antibiotics?

A

Probiotics may reduce the risk of both antibiotic-associated diarrhea more generally, and Clostridioides (Clostridium) difficile diarrhea specifically, when antibiotics are used.

174
Q

what are blackheads and whiteheads comedones in acne?

A

Open comedones - blackheads due to melanin accumulation

Closed comedones- whiteheads

175
Q

What are some topical retinoids?

A

Topical retinoids (adapalene, tretinoin, tazarotene, and trifarotene) are appropriate for the treatment of mild to moderate acne as single agents, although they may be more effective when combined with a topical antibiotic or benzoyl peroxide. Topical antibiotics can lead to bacterial resistance and should not be used as monotherapy. Oral antibiotics are appropriate for the treatment of moderate to severe acne that has failed to respond to topical treatment. Oral isotretinoin is reserved for the treatment of severe nodular acne.

176
Q

What is associated with adhesive capsulitis

A

Diabetes and hypothyroidism have been found to have a prevalence of 25%–50% in patients with adhesive capsulitis (frozen shoulder). Consideration should be given to testing for both of these conditions when making the diagnosis of adhesive capsulitis (level of evidence C).

177
Q

What BP medication is recommended when patients have gout?

A

Losartan is the preferred antihypertensive agent in gout due to it lowers serum urate concentrations. Hydrochlorothiazide increases serum urate concentrations.

178
Q

Urine immunoassay drug screenings can result in false negative result for which drugs?

A

Immunoassay drug screenings can be performed at the point of care and are relatively inexpensive. Typical immunoassays can detect nonsynthetic opioids such as morphine and codeine, as well as illicit substances such as amphetamines, cannabinoids, cocaine, and phencyclidine. They do not reliably detect synthetic or semisynthetic opioids such as oxycodone, oxymorphone, methadone, buprenorphine, and fentanyl, as well as many benzodiazepines. Confirmatory testing is needed in situations with an unexpected negative result in order to distinguish a false negative from a true negative.

179
Q

What to give pregnant women at high risk for preeclampsia?

A

USPSTF recommends prescribing low-dose aspirin after 12 weeks gestation for asymptomatic women at high risk for preeclampsia. Women at high risk include those with a history of preeclampsia, chronic hypertension, multiple pregnancy, type 1 or 2 diabetes, renal disease, autoimmune disease, or any combination of these.

180
Q

When do you treat HTN in pregnant women?

A

Treatment with antihypertensive medications in pregnant patients with chronic hypertension is recommended only when BP is >150/100 mm Hg because aggressive blood pressure lowering may result in placental hypoperfusion.

181
Q

When to start screen for Autism and iron deficiency in children

A

Autism - 18 months

Iron deficiency - 12 months

182
Q

Treatment for symptomatic fibroid

A

Uterine artery embolization and occlusion is effective for reducing fibroid size, with lasting effects up to 5 years and moderate evidence for reducing bleeding and improving quality of life.

183
Q

When to get imaging for back in children and adolescents?

A

Guidelines from the American College of Radiology state that imaging in children and adolescents with back pain can be delayed unless there are abnormal neurologic findings, pain that awakens the patient at night, or pain that radiates or persists for more than 4 weeks.

184
Q

When can child sit in front seat? When can child be restrained in care seat or booster seat?

A

13 yo for front seat.

Children 4–8 years of age may be appropriately restrained in a car seat or booster seat

184
Q

When can child sit in front seat? When can child be restrained in care seat or booster seat?

A

13 yo for front seat.

Children 4–8 years of age may be appropriately restrained in a car seat or booster seat

185
Q

Care for chronic indwelling urethral catheters.

A

Care for chronic indwelling urethral catheters. Prevention of catheter-associated urinary tract infections (CAUTIs) is important. The most important measure to prevent CAUTIs is routine cleaning of the meatal surface with soap and water while bathing or showering. Catheters and drainage bags should only be changed when clinically indicated, such as when there is an infection or obstruction.

186
Q

Describe classes of burns - superficial burns, superficial partial thickness burns, deep partial thickness burns, full thickness burns

A

Superficial burns are red, painful, and blanching, and they do not blister. Superficial partial-thickness burns blister and blanch with pressure. Deep partial-thickness burns blister, but do not blanch with pressure. Full-thickness burns extend through the entire dermis and into the underlying tissues, and they are dry and leathery. Patients with deep partial-thickness or full-thickness burns should be evaluated by a burn specialist.

187
Q

CURB-65 score and treatment outpatient and inpatient

A

CURB-65 score - confusion, BUN
score 0-1, outpatient
score 2, inpatient vs observation admission
score 3 or more, inpatient admission with consideration for ICU with score 4 or 5

Outpatient - amoxicillin/clavulanate (augmentin) plus azithromycin
Inpatient - levofloxacin alone or ceftriaxone plus azithromycin

188
Q

Management for compression fracture of the distal radius

A

Compression fracture of the distal radius (buckle fracture). There is no cortical disruption and these are inherently stable fractures. Radiography or ultrasonography for initial imaging study if a buckle fracture is suspected. Treatment consists of short arm immobilization, which is most easily performed with a removable splint or wrist brace. The Choosing Wisely campaign - does not require repeat imaging if there is no longer any tenderness or pain with palpation after 4 weeks of splinting, and the patient can return to full activity as tolerated. Does not require referral to an orthopedist and can be managed in the office.

189
Q

Treatment for acute rheumatic fever.

A

Acute rheumatic fever - two major criteria (carditis and polyarthritis) and two minor criteria (fever and positive erythrocyte sedimentation rate). Treatment is NSAIDs can provide significant relief.

189
Q

Treatment for acute rheumatic fever.

A

Acute rheumatic fever - two major criteria (carditis and polyarthritis) and two minor criteria (fever and positive erythrocyte sedimentation rate). Treatment is NSAIDs can provide significant relief.

190
Q

Dermatitis herpetiformis is pathognomonic with what disease?

A

Dermatitis herpetiformis is an immunologic response to ingested gluten and is pathognomonic for celiac disease. The papulovesicular rash is extremely pruritic and found on extensor surfaces such as elbows and knees, as well as the buttocks and scalp.

191
Q

Women experience what during menopausal transition.

A

Women experience subjective cognitive difficulties during their menopausal transition. This may include retrieving numbers or words, losing one’s train of thought, forgetting appointments, and forgetting the purpose of behavior such as entering a room. Clinical studies of these women showed intact cognitive test performance. The treatment consists of patient education and reassurance. No trials that support the use of hormone therapy

192
Q

Treatment for subclinical hyperthyroidism

A

Subclinical hyperthyroidism with a low TSH level and normal T3 and T4 levels. The American Thyroid Association recommends observation for asymptomatic patients with mildly low TSH (0.1–0.4 U/mL)

193
Q

What test is specific for Rheumatoid arthritis (RA)?

A

Anti-citrullinated protein antibody is >95% specific for RA when significantly elevated. Rheumatoid factor is nonspecific for RA and may be positive due to cancer, infection, and other autoimmune conditions.

Rheumatoid arthritis (RA) - joint stiffness that lasts 1h in the morning

194
Q

Treatment for COPD

A

Group A - SABA, SAMA, or combination SABA-SAMA as needed.
Group B - LAMA or LABA daily. SABA as needed for symptom relief.
Group C - LAMA daily, SABA as needed.
Group D - LAMA daily or if severe breathlessness, combination LABA plus LAMA. Combination ICS-LABA may be preferred if features of asthma/COPD overlap. SABA as needed.

195
Q

Treatment for alcohol use disorder

A

Naltrexone has been shown to decrease heavy drinking, daily drinking, and the amount of alcohol consumed (SOR A).

196
Q

Syncope is classified into what three broad categories?

A

cardiac
neurally mediated
orthostatic hypotension.

197
Q

Treatment for calcium stones

A

Calcium stones, composed of either calcium oxalate and/or phosphate, account for up to 90% of all stones in adults in developed countries. Increasing fluid intake to 2.5–3 L/day is the most important lifestyle modification to prevent recurrent kidney stones. A diet rich in fiber and vegetables with normal calcium content (1–1.2 g/day), limited sodium intake (4–5 g/day), and limited animal protein intake (0.8–1 g/kg/day) is strongly encouraged. Reduction of BMI is also recommended. Citrate supplementation with Potassium Citrate is recommended for preventing calcium stones that recur despite lifestyle modifications. Thiazide diuretics in higher dosages, such as 50 mg daily of hydrochlorothiazide, have also been shown to be effective in preventing calcium stone formation. Allopurinol is also an effective option. Elimination of all calcium from the diet, such as a low- to very-low-calcium diet, is discouraged as it not only increases stone formation but may also result in bone demineralization. Furosemide increases urinary calcium excretion and would increase the likelihood of calcium stone formation.

198
Q

Adverse effects of testosterone intake for female to male transgender and estrogen for male to female transgender.

A

Female to male transgender - Risks of testosterone therapy include more atherogenic lipid profiles, an increase in blood pressure, and erythrocytosis (rather than anemia).

Estrogen-based therapies for male-to-female transgender patients do carry an increased risk for VTE.

199
Q

Treatment for osteoarthritis of the metatarsophalangeal (MTP) joint

A

Hallux rigidus due to osteoarthritis of the metatarsophalangeal (MTP) joint and presents as decreased range of motion, swelling, and pain. With progression of the condition, flare-ups become more frequent and more severe, and it can be mistaken for gout. Initial treatment is restriction of motion across the MTP joint. A stiffening shoe insert does relieve pain and most patients see improvement without surgery. Custom orthotics, rigid inserts, or hard-soled shoes are options that are more effective than NSAIDs.

200
Q

Leading cause of mortality in people age 15-44, age 45-64, and are 65 or older

A

age 15-44 - accidents
age 45-64 - malignancy
age 65 or older - heart disease

201
Q

Treatment for plantar warts

A

Salicylic acid is effective for the treatment of plantar warts. Duct tape has not been shown to be more effective than placebo.

202
Q

True or False: Cytokine storm is immune dysregulation, not anaphylaxis

A

True. Cytokine storm is caused by the release of cytokines and is characterized by fever, tachypnea, headache, tachycardia, hypotension, rash, and/or hypoxia. Cytokine storm can be triggered by certain therapies, pathogens, cancers, autoimmune conditions, and monogenic disorders. It does not involve histamine release or anaphylaxis.

203
Q

Next step after confirming low testosterone with two morning lab tests.

A

Next step is to determine the cause of the low testosterone. Checking LH and FSH levels is recommended to evaluate for primary hypogonadism. If primary hypogonadism is present, chromosomal studies should be considered. Before initiating testosterone therapy, checking the patient’s PSA level and performing a digital rectal examination are recommended.

204
Q

Describe thoracic outlet syndrome

A

Thoracic outlet syndrome group of disorders that occur when blood vessels or nerves in the space between your clavicle and your first rib (thoracic outlet) are compressed. It can be differentiated into neurogenic, venous, or arterial, with neurogenic being the most common, constituting over 95% of cases. This patient has venous thoracic outlet syndrome, which is the second most common, occurring in about 3% of cases. Swelling of the arm with associated pain strongly suggests obstruction of the subclavian vein. Paresthesias in the fingers and hand are common, likely due to swelling rather than nerve compression at the thoracic outlet. Venous thoracic outlet syndrome is easily identified by swelling, cyanosis, and distention of superficial veins in the arm. Due to the exceptionally high risk of developing a venous thrombosis, patients should undergo diagnostic evaluation with upper extremity venous duplex ultrasonography. False negatives are common in patients without a thrombus and in such cases the patient may benefit from evaluation with either contrast-enhanced upper extremity CT or magnetic resonance venography. If a thrombosis is present anticoagulation should be started immediately and catheterization of the vein should be performed with thrombolysis with or without balloon angioplasty. Ultimately the patient will require surgical decompression.

205
Q

Examples of obstructive vs restrictive pattern PFTs conditions.

A

Obstructive PFT- asthma, COPD, 1-antitrypsin deficiency, bronchiectasis, cystic fibrosis

Restrictive PFT - adverse reactions to nitrofurantoin, methotrexate, and amiodarone. Chest wall conditions such as kyphosis, scoliosis, and morbid obesity. Interstitial lung disease, including idiopathic pulmonary fibrosis, sarcoidosis, and asbestosis, also causes a restrictive pattern (SOR A).

206
Q

What is corrective intervention for metatarsus adductus in infants who are not walking?

A

Adjustable orthotic shoes in infants who are not yet walking can be effective for the treatment of metatarsus adductus (SOR B). These orthotics can be adjusted to apply an abduction force on the forefoot while maintaining the heel in a neutral position. Night splints, braces, and physical therapy are not indicated or proven to correct this deformity. Surgery has high complication rates and is rarely indicated to treat metatarsus adductus.

207
Q

Threshold for BP to treat in acute ischemic stroke?

A

Because patients with an acute ischemic stroke may require the increased perfusion pressure to limit ischemia, antihypertensive therapy should not be given during the first 48–72 hours as long as they are not candidates for, or recipients of, reperfusion therapy with alteplase or thrombectomy; do not have a comorbid condition requiring acute blood pressure lowering; and do not have a blood pressure >220/120 mm Hg.

208
Q

Treatment for anorexia nervosa

A

Psychotherapy is the foundation of treatment and parental involvement is key for children and adolescents. Parents or guardians typically have a high level of distress around their child’s condition and family therapy helps provide consistent support for treatment goals set by the care team. Psychotropic drugs have not been shown to add benefit to psychotherapy, although they are often prescribed.

209
Q

XR would be indicated with what signs according to the Ottawa knee rule?

A

The Ottawa knee rule is a validated tool that decreases unnecessary radiographs in patients with an acutely injured knee. Criteria for imaging according to the Ottawa knee rule include any of the following: age >55, isolated tenderness of the patella, tenderness of the fibular head (lateral), inability to flex the knee to 90°, and inability to bear weight for four steps both immediately after the injury and at the time of the examination. In the absence of these findings patients are highly unlikely to have a clinically significant fracture.

210
Q

Initial treatment for mild dehydration in children. What is BRAT diet.

A

An oral rehydration solution is the treatment of choice for mild dehydration in children with acute gastroenteritis. However, prescribing a formal oral rehydration solution is not necessary. Initial rehydration with diluted apple juice followed by preferred fluids resulted in fewer treatment failures than formal electrolyte solution. Likely due to the increased likelihood that children will drink preferred fluids due to better taste, tolerability, and ease of administration. Intravenous fluids should be reserved for cases of moderate to severe dehydration. BRAT diet is bananas, rice, applesauce, and toast. BRAT diet is a soft bland diet that helps ease GI symptoms of nausea, vomiting, and diarrhea in gastroenteritis.

211
Q

Treatment for acute pericarditis

A

In a patient with acute pericarditis, after determining that the patient is not at high risk for complications, does not have acute myocardial injury, and is an appropriate candidate for outpatient treatment, there are several options for treatment. Any of the NSAIDs alone are effective in many patients, but some patients do not respond sufficiently, so the addition of colchicine would be the treatment of choice. Colchicine alone is also an appropriate initial treatment, but in case of insufficient response to NSAIDs, the combination is the most effective treatment. Corticosteroids are best reserved for pericarditis related to a connective tissue disease, but they are not recommended in viral or idiopathic pericarditis or in pericarditis in patients with post–acute myocardial infarction pericarditis.

212
Q

Treatment for abnormal uterine bleeding in women >45 yo.

A

Due to the increased risk of endometrial cancer, current guidelines recommend that all women >45 years of age presenting with abnormal uterine bleeding undergo endometrial sampling. Transvaginal ultrasonography is recommended if a bimanual examination is abnormal or if symptoms persist despite treatment.

213
Q

Treatment for charcot neuropathy

A

Acute Charcot neuropathy is a commonly missed diagnosis. The diagnosis should be considered in patients over age 40 with neuropathy and obesity who present with unilateral foot swelling. There may be associated erythema and warmth, and pain may be absent. In a patient with suspected acute Charcot neuropathy, bilateral weight-bearing radiographs are recommended to detect fractures of the midfoot.

214
Q

When to start anticoagulation on CHA2DS2-VASc scoring for atrial fibrillation?

A

Direct oral anticoagulants such as apixaban, betrixaban, dabigatran, edoxaban, and rivaroxaban are first-line agents for prevention of stroke in patients with nonvalvular atrial fibrillation with a CHA2DS2-VASc score 2 or higher in men or 3 or higher in women.

215
Q

What supplement should be added to breastfed premature infants after 1 month of life?

A

Breastfed infants born before 37 weeks gestation should receive iron supplementation at 2 mg/kg/day after 1 month of life.

216
Q

Treatment for partner in early syphilis

A

Partner should be treated presumptively for early syphilis, even though the serologic test result is negative, because he had sexual contact within the past 90 days with a person who was diagnosed with secondary syphilis. When the contact occurred more than 90 days before confirmation of a negative serologic test result, no treatment is necessary. Treatment for primary, secondary, or early latent syphilis is a single dose of penicillin G benzathine, 2.4 million units.

217
Q

Classify the etiology for different areas of ulcers.

A
  • Ulcers due to peripheral neuropathy. Peripheral neuropathy can predispose patients to abnormal gait patterns and/or unrecognized trauma. These deep ulcers usually present over a bony prominence and are surrounded by a callus (SOR A). Example is bottom of foot.
  • Arterial origin ulcers are due to tissue ischemia and are most typically deep but on the anterior leg, distal dorsal foot, and toes, and have a dry, fibrous base with poor granulation tissue. Tendons can be exposed.
  • Venous ulcers are due to venous hypertension and chronic venous insufficiency. These ulcers are shallow and exudative with good granulation tissue in the base. Common locations are over bony prominences such as the medial malleolus.
  • Pressure ulcers occur on areas of high pressure in patients with limited mobility, especially on the sacrum, heels, and hips.
218
Q

What helps to reduce risk of diverticulitis recurrence.

A

Risk factors for diverticulitis include low dietary fiber, a sedentary lifestyle, obesity, and smoking. Avoidance of nuts, seeds, and corn has not been shown to decrease risk for diverticular disease, including diverticulitis.

219
Q

What is Takotsubo syndrome (acute stress-induced cardiomyopathy) and clinical presentation?

A

Takotsubo syndrome (acute stress-induced cardiomyopathy) is transient wall motion abnormalities on echocardiography, usually following an emotionally triggering event. Clinical presentation of stress cardiomyopathy is similar to that of an acute coronary syndrome. Although coronary artery disease may be present, and the two diagnoses may be seen together, coronary artery disease is seen in only about 15%, so coronary angiography is more likely to be normal than the other listed diagnostic tests. Cardiac biomarkers such as CK-MB and troponin are usually elevated, but to a lesser degree than with an acute myocardial infarction. An abnormal EKG is found in >95% of patients with Takotsubo syndrome.

220
Q

Diet for geriatric patients with aspiration risk.

A

This patient is at risk for aspiration pneumonia due to his neurologic disease and impaired cough reflex. A swallow evaluation is appropriate. A mechanical soft diet with thickened liquids is recommended rather than pureed foods and thin liquids.

221
Q

Conditions associated with koilonychia (spoon nail, concave nail) and nail clubbing.

A

Iron deficiency anemia is the most common cause of koilonychia (spoon nail). It appears as a central depression in the nail that curves outward away from the nailbed, giving the nail the appearance of a spoon. If iron deficiency anemia is the cause of koilonychia, the nail will return to a normal appearance when the anemia is corrected. Chronic pulmonary disease is associated with clubbing of the nails.

222
Q

What is fasting BG goals in gestational diabetes? What medication to start if not at goal?

A

Pharmacologic treatment should be initiated in patients with gestational diabetes mellitus (GDM) when nutrition and exercise therapy are not adequate to meet goals. Accepted goals are fasting blood glucose levels <95 mg/dL, 1-hour postprandial glucose levels <140 mg/dL, and 2-hour glucose levels <120 mg/dL. Although oral antidiabetic medications are being used more frequently in GDM, INSULIN is the preferred treatment recommended by the American Diabetes Association and the American College of Obstetricians and Gynecologists. Oral medication may be initiated in patients who refuse insulin or are unable to comply with insulin management. This recommendation is made predominantly because metformin has not shown superiority and there is a lack of long-term outcome studies in the offspring exposed to metformin.

223
Q

Treatment for trigger finger

A

Several options for trigger finger with conservative treatment prior to consideration of surgical release. Splinting, which is a first-line treatment, has been shown to be effective. The duration of splinting can range from 6 weeks to 3 months. A retrospective case series analysis of trigger finger managed by observation only found that trigger finger resolved spontaneously in 52% of patients, with the majority resolving within 1 year. Surgical release is considered the most effective treatment but not the most cost-effective.

224
Q

Medications that can cause hypertriglyceridemia.

A

Several medications can cause hypertriglyceridemia, including BETA-BLOCKERS, with the exception of carvedilol. Others include oral estrogens, glucocorticoids, bile acid sequestrants, protease inhibitors, retinoic acid, anabolic steroids, sirolimus, raloxifene, tamoxifen, and thiazides.

225
Q

Treatment for croup (barking cough).

A

Corticosteroids should be used in the treatment of croup regardless of the degree of severity. Dexamethasone is preferred because it can be given in a single dose and administered either orally, parentally, or intravenously. Nebulized epinephrine should be reserved for patients with moderate to severe croup. Oxygen should be administered if there are signs of hypoxemia or severe respiratory distress.

226
Q

Treatment for PTSD

A

Posttraumatic stress disorder (PTSD). Individual trauma-focused psychotherapy is the intervention that demonstrates the most significant benefit. Pharmacotherapy may be used if psychotherapy is not effective or available. Recommended options include fluoxetine, paroxetine, venlafaxine, or sertraline.

227
Q

What serum marker rules out SLE?

A

The diagnosis of systemic lupus erythematosus (SLE) can be difficult and is often not established for months or even years, due to the significant overlap of symptoms with many other conditions. The American College of Rheumatology has established 11 diagnostic criteria, at least 4 of which must be met over time, to establish a diagnosis of SLE. The vast majority (>95%) of patients with SLE have a positive antinuclear antibody (ANA) test, thus it is sensitive as an initial test in a patient for whom there is clinical suspicion for SLE. Testing for other immunologic subgroup ANA markers should be performed in a patient with a positive ANA. If one or more of those are positive, then the likelihood of SLE is higher. The majority of patients with a positive ANA do not have SLE but a negative ANA is very unlikely in a patient who has SLE. The subgroup markers (anti-dsDNA, anti-SmDNA, complement C3, C4, CH50) should only be obtained in patients suspected of having SLE who have a positive ANA.

228
Q

Management of scoliosis in children

A

Adolescent idiopathic scoliosis is generally defined as a lateral curvature of the spine or Cobb angle greater than or equal 10°. Cases with a Cobb angle <20° can generally be managed with observation. Surgical evaluation is reserved for severe cases or those with a Cobb angle greater than or equal 40°.

229
Q

Mechanisms of these medications.

A
  • Varenicline is a nicotinic receptor partial agonist, works to reduce nicotine withdrawal by activating the nicotine receptor and producing about 50% of the effect of nicotine. It also prevents tobacco smoke nicotine from binding to the receptor.
  • Bupropion is a norepinephrine-dopamine reuptake inhibitor that reduces nicotine withdrawal and the reward from tobacco smoking.
  • Naltrexone is a pure opioid receptor antagonist that is effective for the treatment of alcohol use disorder and opioid use disorder.
230
Q

First line treatment for prevention of migraines?

A

Many medications have been studied for the prevention of migraine. Divalproex, topiramate, metoprolol, propranolol, and timolol have been shown to be effective for migraine prevention. One of these medications should be offered as first-line treatment.

231
Q

Medications to withhold prior to contrast administration to prevent contrast induced nephropathy.

A

In order to prevent contrast-induced nephropathy, NSAIDs such as naproxen should be withheld for 24–48 hours prior to a procedure involving venous or arterial administration of radiocontrast material. Avoidance of volume depletion and other nephrotoxic agents is also recommended. Pre- and postprocedural hydration with normal saline is recommended in patients at high risk for developing contrast-induced nephropathy, such as those with underlying chronic kidney disease, heart failure, proteinuria, sepsis, hypovolemia, or hypotension. Metformin does not cause contrast-induced nephropathy but should be withheld due to the potential, mostly theoretical, risk of developing lactic acidosis, especially if contrast-induced nephropathy were to develop (SOR B).

232
Q

Most common cause of pulmonary hypertension

A

It is important to determine the etiology since addressing the underlying condition is the preferred treatment for most cases of non-severe pulmonary hypertension. Left heart disease, including both preserved and reduced systolic function, is the most common cause of pulmonary hypertension.

233
Q

EKG changes with acute hyperkalemia

A

EKG changes noted with hyperkalemia include peaked T waves, flattened P waves, PR prolongation, a widened QRS complex, sine waves, sinus bradycardia, ventricular tachycardia, ventricular fibrillation, and asystole. Treatment includes intravenous calcium chloride 10% solution, 10 mL (level of evidence C).

234
Q

Threshold for platelet transfusion who is not currently bleeding?

A

The threshold for transfusing platelets to prevent spontaneous bleeding in the setting of hypoproliferative thrombocytopeniainmostadultsis <10,000/microL(SORA).

235
Q

Differerentiation in estrogen, LH, FSH levels in primary ovarian insufficiency and congenital adrenal hyperplasia.

A

Primary ovarian insufficiency is associated with low estradiol levels and high levels of LH and FSH. Generally, the LH/FSH ratio is <1. Patients with congenital adrenal hyperplasia will have low estrogen, LH, and FSH levels. Virilization is generally noted in congenital adrenal hyperplasia, and a 17-hydroxyprogesterone level should be obtained to assess for this condition. Functional hypothalamic amenorrhea will also cause low levels of LH, FSH, and TSH.

236
Q

Next step in patient with dyspepsia (epigastric pain, bloating, occasional vomiting after eating. No acid reflux).

A

Patients with dyspepsia but no alarm symptoms such as weight loss, blood in the stools, or difficulty swallowing. An important cause of dyspepsia is gastric infection with Helicobacter pylori. Test for H. pylori and treat if positive.

237
Q

Adverse effect of megestrol in patients getting chemo?

A

Megestrol increases the risk of venous thromboembolic events in patients with cancer who are receiving chemotherapy (SOR C).

238
Q

When is latest you can give HPV vaccine

A

CDC recommends HPV vaccine as a routine vaccination for all patients starting at 11 or 12 years of age through 26 years of age but can also be considered in adults 27–45 years of age who have not previously received the vaccine and are most likely to benefit. Meningococcal polysaccharide conjugate vaccine is not routinely recommended for patients 24 years of age or older.

239
Q

Treatment for hirsutism (excessive hair growth)?

A

Hirsutism affects 5%–15% of women and can adversely affect quality of life. It is caused by increased androgen production. Most cases are caused by benign conditions. Polycystic ovary syndrome (PCOS) accounts for 70% of cases with another 25% attributable to idiopathic hyperandrogenism and idiopathic hirsutism. First-line therapy for hirsutism in women who do not desire pregnancy is combined oral contraceptives (SOR B), which decrease androgen production in the ovaries by decreasing LH levels.
Antiandrogen treatments such as spironolactone and finasteride are second-line therapies that can be added to the combined oral contraceptives if there is no improvement after the first 6 months (SOR A).

240
Q

Signs of hypocalcemia vs hypercalcemia

A

A Trousseau sign (carpopedal spasm = painful cramps of the muscles in your hands and feet), defined as spasmodic contraction of muscles caused by pressure on the nerves that control them, is present in up to 94% of patients with hypocalcemia. Hypercalcemia is more likely to present with hyperreflexia.

241
Q

Next step in superior vena cava syndrome

A

Signs and symptoms of superior vena cava syndrome (facial swelling, progressive dyspnea), which is caused by compression of the superior vena cava. This is most often caused by lung cancer or lymphoma, but it can also be related to indwelling catheters, lymph nodes, or metastatic tumors. After ensuring that the patient is hospitalized and stable, the initial treatment options include intravenous corticosteroids, chemotherapy, radiation, and occasionally intravascular stenting.

242
Q

Define stridor and next step.

A

Stridor is a high-pitched whistling, crowing sound on inspiration. It can be caused by obstruction of the larynx or trachea by a foreign body, vocal cord edema, a neoplasm, or a pharyngeal abscess. Acute stridor requires urgent evaluation for obstruction. This patient may have a foreign body or other obstruction in his airway and requires urgent assessment in the ED.

243
Q

Patients with Heparin-induced thrombocytopenia (HIT) are at increased risk of what?

A

Heparin-induced thrombocytopenia (HIT) is an immune-mediated process that occurs in approximately 1 in 5000 hospitalized patients. Patients are at highest risk 7–10 days after exposure to unfractionated heparin, and the risk is particularly high after cardiac surgery, which is associated with an estimated rate of 1%–3%. HIT places patients at a paradoxically increased risk of thrombotic complications, with clotting events occurring in roughly 50% of confirmed cases of HIT. Lower-extremity deep vein thrombosis and pulmonary embolism are the most common thrombotic complications, followed by arterial thromboses, stroke, and myocardial infarction, in descending order of frequency. Thromboses often occur concurrently with the development of thrombocytopenia or shortly thereafter.

244
Q

Describe the four classes of NYHA heart failure classes.

A

New York Heart Association (NYHA) functional classification in HF.
Class I - no symptoms and no limitations of physical activity.
Class II - have mild symptoms with normal physical activity.
Class III - significant limitations of activity, including symptoms with less than normal activities
Class IV - have symptoms at rest and are unable to carry on activity without discomfort.

245
Q

Treatment for recurrent Bartholin gland abscess

A

Treatment for recurrent Bartholin gland abscess would be marsupialization, which has a 0% recurrence rate at 6 months. Marsupialization is a surgical procedure that removes cysts in a way that makes them less likely to return. Once the cyst has been opened and drained, the edges are sutured together to form a permanently open “pocket” or “pouch” that allows fluid to drain easily. Local anesthesia can be used in the office to effectively treat Bartholin gland abscesses and sedation is not required (SOR A). If the Bartholin gland abscess is >5 cm, referral to a gynecologist is recommended.

246
Q

Abortive migraine treatment in pregnancy

A

Metoclopramide and acetaminophen are the only two medications considered safe for abortive migraine treatment during pregnancy (SOR B). The dopamine antagonist antiemetics are considered second-line abortive treatments in the general population. NSAIDs are not considered safe during pregnancy, particularly in the first and third trimesters. Triptans are generally considered safe during the first trimester but not in the second and third trimesters. Their use has been associated with uterine atony, increased risk of bleeding during delivery, and increased risk of preterm birth.

247
Q

Give examples of ADLs and iADLs

A

The foundation of geriatric assessment is assessing the individual’s ability to perform tasks required for living. Activities of daily living (ADL) are self-care activities that are performed daily - eating, bathing, dressing, transferring between the bed and a chair, and toileting, including bladder and bowel function. Instrumental activities of daily living (iADL) include activities necessary to live independently - using a telephone, doing housework, preparing meals, taking medications properly, and managing finances.

248
Q

What are treatment in children with acute asthma exacerbation if does not improve with SABA and steroids?

A

Children who present to the emergency department with an asthma exacerbation and fail to improve adequately with inhaled short-acting bronchodilators and corticosteroids may benefit from treatment with intravenous (IV) magnesium sulfate. A 2016 Cochrane review of three randomized, controlled trials found that this reduced hospital admissions by 68%.

249
Q

How to remove impacted cerumen in patients

A

Cerumen impaction resulting in hearing loss can cause reversible cognitive impairment in older persons with dementia. Treatment options include irrigation with warm water, cerumenolytic agents such as carbamide peroxide otic, or manual removal if the patient is cooperative and if the procedure can be completed without the use of restraints.

250
Q

Treatment for herpetic whitlow

A

Herpetic whitlow is a viral infection of the distal finger caused by herpes simplex. Primary herpetic whitlow is generally a self-limited infection. The recommended treatment is pain management and keeping it covered with a dressing to prevent transmission. Off-label use of antiviral medications should be considered only for patients with recurrent lesions, those with symptoms for less than 48 hours, and those who are immunocompromised.

251
Q

Difference between digital mucous cyst vs digital wart?

A

Digital mucous cyst (cutaneous myxoid cyst). Mucous cysts most commonly occur on the dorsal surface of the distal phalanx. Toe lesions are less common. The etiology is controversial. Treatment options include intralesional corticosteroid injections, repeated puncture and drainage, or surgical excision (SOR A). Wart has a verrucous texture and appearance.

252
Q

Turner syndrome as etiology for delayed puberty.

A

Family physicians are often asked to evaluate delays in puberty. Underlying etiologies should be excluded in females >13 years of age who lack any breast development, which may signify delayed puberty. A past medical history and a physical examination, as well as a gonadotropin measurement, should be performed. Females with TS lack normal X chromosome gene expression and typically have delayed puberty; amenorrhea; elevated FSH, reflecting hypogonadism; and short stature. Delayed diagnosis of TS is common, and short stature and delayed puberty are sometimes the only symptoms. This patient has unexplained short stature, delayed puberty, and an elevated FSH level, so karyotyping to rule out TS is the next step in evaluation.

A corticotropin stimulation test rules out Cushing syndrome in a setting of precocious puberty.

253
Q

Define COPD PFTs

A

COPD is irreversible obstructive pulmonary defect with FEV1 <80% of predicted and an FEV1/FVC ratio <0.70

254
Q

Define alpha-1-antitrypsin deficiency

A

alpha-1-antitrypsin deficiency - Patient has both an irreversible obstructive pulmonary defect consistent with COPD (FEV1 <80% of predicted and an FEV1/FVC ratio <0.70) and liver abnormalities associated with advanced fibrosis (low platelets, elevated AST/ALT, low albumin)

255
Q

Benefits of exercise

A

Exercise alone does have some substantial benefits, including improved insulin and glycemic control in diabetes, a beneficial effect on blood pressure, a reduction of cardiovascular risks, and a maintenance of weight loss. However, it is only moderately beneficial for promoting weight loss, including when exercise is added to diet changes.

256
Q

When to use DOAC vs warfarin in atrial fibrillation

A

According to 2019 guidelines from the American College of Cardiology, American Heart Association, and Heart Rhythm Society, patients with nonvalvular atrial fibrillation and an elevated CHA2DS2-VASc score (2 or more in men and 3 or more in women) should receive anticoagulation, preferably with a direct-acting oral anticoagulant (DOAC), rather than warfarin (SOR A). DOAC options are not inferior to, and in some studies are superior to, warfarin for preventing strokes and systemic embolic events, with a lower risk of serious bleeding. However, use warfarin for valvular atrial fibrillation that occurs in the presence of moderate to severe mitral stenosis or a mechanical heart valve.

257
Q

The most common symptom of alcohol withdrawal in the elderly is

A

In older adults the onset of alcohol withdrawal syndrome may not occur until several days after the cessation of drinking. Confusion, rather than tachycardia or tremor, is often the predominant clinical sign, and the severity and duration of withdrawal tend to increase with age. Alcohol withdrawal should be considered as a cause of confusion in older patients and may be manifested as new-onset confusion in a hospitalized older patient.

258
Q

Auscultatory finding in aortic regurgitation, aortic stenosis, and mitral regurgitation

A

Aortic regurgitation in an older adult may be due to a congenital bicuspid aortic valve, which often is accompanied by aortic stenosis. Rheumatic aortic valvular disease may also cause aortic regurgitation, which is the most common cause in the developing world but less common in the United States.
The hallmark murmur of aortic regurgitation is a “blowing” decrescendo diastolic murmur along the tract from the aortic valve (upper right sternal border) down to the lower left sternal border, where it is loudest. It is best heard with the patient sitting, leaning forward, and holding his or her breath in expiration.
Both bicuspid aortic valve and rheumatic valve disease may also be associated with aortic stenosis. The murmur of aortic stenosis is a mid- to long crescendo/decrescendo systolic murmur, loudest at the right upper sternal border, and often radiating to the carotid arteries.
A harsh holosystolic murmur at the lower left sternal border radiating to the axilla is characteristic of mitral valve regurgitation, another potential condition found in rheumatic valvular disease.
Pericardial friction rub is the principal auscultatory finding in acute pericarditis.

259
Q

Which oral iron is most effective for IDA in non-dialysis dependent CKD?

A

Oral ferric citrate is highly efficacious in patients with non–dialysis-dependent chronic kidney disease or on hemodialysis. IV iron dextran are similarly effective in both groups and are considered the gold standard for the treatment of iron deficiency in patients on hemodialysis.

260
Q

Describe medicare part A, B, and D.

A

Medicare Part B covers provider visits and outpatient services such as laboratory testing. Beneficiaries are automatically enrolled in Part A when they apply to Medicare, which provides coverage for hospital-based and hospice care. Because Part A does not typically carry a monthly premium, some working older adults who continue to have insurance through their employer may opt to obtain Part A only, and wait on Part B coverage, which does have a monthly premium. Medicare Part D is prescription drug coverage.

261
Q

Define the Ottawa knee rule for obtaining XR.

A

The Ottawa knee rule is a validated tool that decreases unnecessary radiography in patients with a knee injury. According to the Ottawa knee rule, a radiograph should be obtained if any of the following are present:
• age 55 or more yo
• isolated tenderness of the patella
• tenderness of the head of the fibula
• inability to flex the knee to 90°
• or the inability to bear weight for four steps both immediately after the injury and at the time of the examination

262
Q

What condition is associated with polymyalgia rheumatica (PMR)?

A

Both polymyalgia rheumatica (PMR) and giant cell arteritis are chronic inflammatory diseases. PMR is the most common chronic inflammatory condition in older adults. Giant cell arteritis is common in patients with PMR. Giant cell arteritis can affect any medium or large artery, particularly the extracranial carotid branches. The temporal artery is commonly involved, and the ophthalmic artery may also be affected. This can result in neuro-ophthalmic complications, including permanent blindness. For this reason giant cell arteritis is considered a medical emergency and it is essential for family physicians to evaluate any patient with PMR for giant cell arteritis.

263
Q

What follow up blood test to check in colorectal cancer survivors?

A

The Choosing Wisely campaign recommends checking only carcinoembryonic antigen (CEA) levels following curative treatment for colorectal cancer (SOR C).

264
Q

What is most effective in preventing progression from Prediabetes to diabetes?

A

Patients who participate in long-term lifestyle intervention programs such as the CDC’s National Diabetes Prevention Program have an approximately 30% reduction in progression to type 2 diabetes. These programs promote weight loss of 7% of body weight and encourage physical activity with a weekly goal of 150 minutes of moderate-intensity exercise. Prediabetes is diagnosed by a fasting glucose level of 100–125 mg/dL, a hemoglobin A1c of 5.7%–6.4%, or a 2-hour plasma glucose level of 140–199 mg/dL.
Metformin is also effective in reducing the progression to diabetes, but it is not as effective as lifestyle intervention programs for most patients.

265
Q

Tobacco cessation in pregnancy does what to infant?

A

Smoking during pregnancy increases the risk for fetal growth restriction. Tobacco cessation increases infant birth weight and decreases risk for preterm delivery.

266
Q

Describe signs of pulmonary aspergillosis

A

Pulmonary aspergillosis comprises a spectrum of clinical disease, from invasive, often cavity-forming disease in critically ill and profoundly immunosuppressed patients to allergic bronchopulmonary aspergillosis. This case of poorly controlled asthma associated with eosinophilia and bronchiectasis is typical of allergic bronchopulmonary aspergillosis. Aspergillus IgE titers are recommended as initial testing in patients with suspected allergic bronchopulmonary aspergillosis. Antifungal treatment can improve outcomes in these cases.

267
Q

Describe de Quervain’s tenosynovitis

A

This patient has de Quervain’s tenosynovitis, caused by inflammatory changes in the extensor pollicis brevis and the abductor pollicis longus or their tendon sheaths. It should be suspected when a patient presents with insidious pain in the radial wrist at the base of the thumb. Some patients may report a history of new repetitive use of the hand. It is more common in women, especially new mothers who are repeatedly picking up their children. The Finkelstein test, which involves making a fist over the thumb and moving the hand into ulnar deviation, has good sensitivity and specificity and is usually sufficient to make the diagnosis.

268
Q

Treatment for frostbite

A

Frostbite is a freezing injury that occurs when initial cooling causes vasoconstriction and localized ischemia. Continued cold exposure leads to ice crystal formation, which causes cellular lysis, electrolyte abnormalities, and microvascular occlusion. Rewarming creates an inflammatory response. Ibuprofen is the most appropriate agent for the treatment of frostbite until the wounds heal or surgery is performed (SOR C).

269
Q

Treatment for vertebral compression fracture

A

The differential diagnosis for acute nonradicular low back pain is broad and should include osteoarthritis, discitis, myofascial pain, and vertebral compression fracture, among other possibilities. The most appropriate next step in vertebral compression fracture is to proceed with conservative pain management with scheduled acetaminophen and topical lidocaine patches. Significant controversy persists regarding the use of percutaneous vertebral augmentation (vertebroplasty or kyphoplasty) for vertebral fractures, in large part because studies suggests no benefit to the procedures over placebo.

270
Q

What condition is command seen in non-atopic asthma.

A

This patient presents with adult-onset, severe and persistent, non-atopic asthma and aspirin sensitivity. This type of asthma is defined as intrinsic asthma and affects approximately 10% of patients with asthma. Patients with intrinsic asthma commonly also have nasal polyps. Allergic rhinitis and eczema are both associated with atopy and patients with these conditions would also have positive skin tests for inhalant allergens and elevated IgE levels.

271
Q

Next step in thyroid nodule

A

TSH first, then thyroid US. Thyroid nodules 1 cm or greater that are solid or have suspicious features require a fine-needle aspiration biopsy to rule out malignancy. Fine-needle aspiration should not be performed on nodules <1 cm.

272
Q

Patients with egg allergy can receive what form of the influenza vaccine?

A

It is important to provide influenza vaccine to as many individuals as possible. Patients with an egg allergy can receive any form of the influenza vaccine. Patients who have an angioedema reaction to egg products should receive the vaccine in an office setting. Testing for specific responses to egg allergens is not recommended prior to giving the vaccine.

273
Q

What labs indicate cirrhosis and what medication to avoided in cirrhosis?

A

Unrecognized compensated hepatic cirrhosis. While the diagnosis of cirrhosis should be confirmed and assessed by methods such as transient elastography, the family physician should recognize this as presumed cirrhosis based on the splenomegaly and laboratory findings (low platelets, elevated FLTs, low albumin). NSAIDs should be avoided in patients with cirrhosis due to the risk of renal insufficiency (SOR B). While toxic to the liver at high doses, acetaminophen can be safely used for analgesia in cirrhotic patients, though many hepatologists recommend limiting dosing to 2 g daily (SOR C).

274
Q

What type of hallucinations is associated with Parkinson’s disease?

A

Visual hallucinations are associated with Parkinson’s disease, and are seen with Parkinson’s dementia, which is a type of Lewy body dementia. The combination of auditory and visual hallucinations is associated with schizophrenia. Hypnagogic and hypnopompic hallucinations are associated with sleep disorders.

275
Q

Treatment of displaced scaphoid fracture

A

Scaphoid fractures have a high risk for nonunion because the blood supply arises distally from branches of the radial artery. The proximal pole of the scaphoid is entirely dependent on this distal blood supply. To improve healing and decrease the risk of nonunion and avascular necrosis, displaced fractures should be treated with surgical fixation. Nondisplaced fractures of the distal third of the scaphoid may be treated with a short arm thumb spica cast for 4–6 weeks. Middle and proximal fractures should be treated with a long arm thumb spica cast for 6 weeks, followed by a short arm thumb spica cast.

276
Q

Treatment for idiopathic hirsutism in women

A

Benign idiopathic hirsutism. It is estimated that approximately 50% of women with mild hirsutism have idiopathic hirsutism. In the absence of other worrisome findings on the history or examination, such as a rapid onset, virilization, or a high degree of hirsutism, the most appropriate next step is a trial of pharmacologic therapy, using oral contraceptive pills as the first-line agent if the patient does not desire pregnancy. A minimum 6-month trial is needed because of the length of the hair growth cycle. OCPs work by suppressing LH and FSH, leading to a decrease in ovarian androgen production.

277
Q

Treatment for hypertensive urgency?

A

This patient has a hypertensive urgency, defined as symptomatic acute severe hypertension without evidence of acute end-organ injury. Hypertensive urgencies may be managed in the ambulatory setting. Treat with an oral agent with a fairly rapid onset of action, such as clonidine, labetalol, captopril, or prazosin. Topical nitroglycerin is also an option.

278
Q

Proximal muscle weakness condition?

A

This patient exhibits classic symptoms and signs of proximal muscle (shoulder and hip girdle musculature) weakness but not distal muscle weakness. Various types of myositis, including polymyositis, are associated with proximal but not distal or bulbar (lower cranial nerves) weakness. Patients with proximal muscle weakness have difficulty walking up or down steps, or they exhibit weakness with overhead lifting or other activities requiring the upper arm and shoulder musculature.
Guillain-Barré syndrome develops fairly rapidly, in a matter of days, and weakness begins distally, then progresses proximally (ascending paralysis), and is also associated with respiratory muscle and bulbar involvement. The neuromuscular manifestations of multiple sclerosis are widely varied, have a gradual onset, are most often unilateral, and usually include bulbar and/or sensory involvement.

279
Q

Which contraceptive method does not require the use of backup contraception?

A

A copper-containing IUD may be placed at any time during the menstrual cycle and does not require the use of backup contraception. If oral contraceptives will be started with the next menstrual cycle, then backup contraception is needed until the pills are started. Backup contraception is needed if a hormonal implant is placed more than 5 days from the start of the last menstrual period, and if a levonorgestrel IUD is placed more than 7 days from the start of the last menstrual period.

280
Q

What test and when to test for clearance of H. pylori after treatment?

A

After treatment for a Helicobacter pylori infection it is essential to document clearance of the infection. Do a stool antigen test or a urea breath test 1 month after the completion of antibiotic therapy. If the patient is taking a proton pump inhibitor it should be discontinued prior to the test.

281
Q

Treatment for trigger finger.

A

Trigger finger is a common reason for referral to a hand surgeon. Risks factors for this condition include trauma, overuse, diabetes mellitus, and carpal tunnel syndrome. It is much more common in women than in men and the average age of onset is 58. Trigger finger develops when there is scarring and inflammation of the A1 pulley, the first of a five-pulley system in the hand. Stenosis of the A1 canal or nodules on the tendon can produce locking, cracking, and pain when the digit is flexed. The most cost-effective treatment strategy is the use of corticosteroid injections. The success rate is 57% after the initial injection and 86% following the second injection within a 6-month time frame. When the problem is mild, NSAIDs and splinting may be effective.

282
Q

Next step in positive hepatitis C antibody screen.

A

The CDC recommends that all adults 18 years of age or older receive a one-time screening for hepatitis C virus (HCV). If the anti-HCV antibody screen is positive, then a qualitative HCV RNA test is the next step (SOR C). The “qualitative” test is more accurate than the “quantitative” test because qualitative tests are able to detect very low levels of the virus. Prior to initiating treatment, a quantitative HCV RNA and genotype testing is necessary (SOR A). In addition, assessing the degree of fibrosis will provide information regarding the urgency of treatment. Percutaneous liver biopsy is generally the preferred evaluation after obtaining quantitative and genotype results, all of which can guide treatment decisions.

283
Q

General diet recommendations after bariatric surgery

A

Many bariatric surgery procedures create a small stomach pouch. Dietary compliance is essential to minimize feeding intolerance symptoms such as postprandial nausea, emesis, and diarrhea. Post bariatric surgery diet recommendations typically include the following:

  • Avoid fluid 15 minutes before and after meals. Fluids with meals will move food more quickly through the pouch and decrease the feeling of fullness.
  • Avoid carbonated beverages entirely.
  • Eat three small protein-rich meals and one or two snacks daily. Lower fat diets are not typically recommended.
  • Whole grains and fibrous vegetables often exacerbate symptoms so there is no need to increase these foods.
284
Q

Treatment for degenerative meniscal tears in adults

A

Arthroscopic procedures for degenerative meniscal tears in middle-aged adults with little or no arthritis do not significantly improve long-term pain or function compared to conservative management consisting of physical therapy and a standardized exercise program.

285
Q

Describe the STOP-Bang questionnaire

A

The STOP-Bang questionnaire is a screening tool to help identify patients with obstructive sleep apnea. In the questionnaire S = snoring, T = tiredness, O = observed apnea, P = high blood pressure, B = BMI >35 kg/m2, A = age >50 years, N = neck circumference >40 cm, and G = male gender. For each question, answering “yes” scores 1, answering “no” scores 0, and the total score can range from 0 to 8, with a higher score indicating a higher probability of obstructive sleep apnea.

286
Q

Use of anticholinergic medications and other sedatives has been prospectively linked to an increased risk of what condition?

A

The use of anticholinergic medications and other sedatives has been prospectively linked to an increased risk of dementia. Treatment of overactive bladder and depression could include anticholinergics,

287
Q

Next step in thyrotoxicosis (thyroid storm).

A

Hyperthyroidism is a common condition with a generally favorable prognosis. However, it is important to remember that life-threatening complications such as thyrotoxicosis, also known as thyroid storm, can occur. Symptoms of thyroid storm include fever, CNS dysfunction, GI or liver dysfunction, and cardiovascular complications such as tachycardia and heart failure. This acute, life-threatening condition typically requires care in an intensive-care unit. It would therefore be inappropriate to start treatment with an agent such as methimazole prior to hospitalization.

288
Q

Treatment for first episode of C. difficile and first recurrent episode.

A

Clostridioides (Clostridium) difficile infection. Initial episodes can be treated with vancomycin (strong recommendation, high quality of evidence), fidaxomicin (strong recommendation, high quality of evidence), or metronidazole if the other two treatments are unavailable (weak recommendation, high quality of evidence). However, fidaxomicin is recommended for recurrent infection if vancomycin was prescribed for the initial episode (weak recommendation, moderate quality of evidence).

289
Q

Treatment of toe phalanx fracture

A

This patient has a minimally displaced oblique right fifth proximal phalanx shaft fracture. Lesser toe fractures such as this typically can be managed conservatively with buddy taping and a rigid-sole shoe. Because this patient has already tried buddy taping, it is appropriate to have him use a rigid-sole shoe. Metatarsal shaft fractures are often initially treated with a posterior splint and then transitioned to a walking cast. Referral to an orthopedic surgeon is typically limited to patients with a high-risk fracture such as a displaced Jones fracture, or to patients who are highly competitive athletes.

290
Q

Treatment for female pattern hair loss (FPHL)

A

Hair thinning on the crown of the head with the presence of small, wispy hairs among the regular hair is characteristic of female pattern hair loss (FPHL). A family history of similar issues is often present but not necessary for the diagnosis.
Topical minoxidil is the mainstay of treatment for FPHL (SOR A). It is available in a 2% solution or 5% foam for women (the 5% solution is indicated only for men). Treatment for FPHL as well as the male equivalent, androgenic alopecia, must be continued long term. With treatment there is often an initial period of increased hair loss. Regrowth is noticeable around 6 months. Discontinuation of treatment results in loss of regrown hair.
There is no clear association between hormone status and FPHL

291
Q

Treatment for hand-foot-and-mouth disease

A

Hand-foot-and-mouth disease is frequently caused by enterovirus 71 or coxsackievirus A16. It is most common in children under 5 years of age and occurs most often in the fall and spring. It is characterized by painful maculopapular or papulovesicular lesions on the hands and feet, and in the oral cavity. Lesions can also appear on the genitals, trunk, or cheek. Management includes symptomatic treatment of pain and oral hydration. Antibiotics and antiviral treatment are not recommended. Laboratory testing is not appropriate for this condition.

292
Q

IV drug users can have what lung condition

A

Although persons who inject drugs are at high risk for a variety of pulmonary infectious diseases, this patient’s presentation, including the relatively slow development of symptoms, is most consistent with pulmonary foreign body granulomas. These result from the injection of crushed pills, talc, or other foreign substances, which are then deposited in the vasculature of the lungs. Adenocarcinoma is not as likely given the patient’s age and nonsmoking history. Branching hyphae would be seen in aspergillosis but this patient does not have fevers or malaise. Caseating granulomas are seen in tuberculosis, which is less likely given the absence of fever and hemoptysis. Noncaseating granulomas, seen in sarcoidosis, would not be more likely in this patient than in the general population.

293
Q

Treatment for bronchiolitis

A

Bronchiolitis is a common lower respiratory tract infection in young children and infants. Respiratory syncytial virus (RSV) is the most common cause. Supportive care with hydration and maintenance of oxygen saturation is important in the treatment of RSV bronchiolitis. Infants with respiratory rates >60/min are often unable to manage oral hydration due to the risk of aspiration. In these cases, intravenous or nasogastric feeds are acceptable. An oxygen saturation >90% is sufficient in RSV bronchiolitis and use of supplemental oxygen to achieve higher levels of oxygen saturation may prolong hospital stays. There is no clear advantage to deep nasal suctioning, which may also be associated with prolonged hospital stays. Routine nasal suctioning is indicated. Bronchodilators are not recommended in the treatment of RSV (level of evidence A). Antibiotics are only indicated with a confirmed bacterial co-infection (level of evidence B). Systemic corticosteroids have shown no benefit in the treatment of bronchiolitis.

294
Q

Treatment for clinical diagnosis of mild diverticulitis

A

In patients with mild diverticulitis, outpatient management with rest and oral fluids is preferred. Avoidance of seeds, nuts, and popcorn does not reduce recurrence rates. CT of the abdomen may be indicated if the diagnosis is uncertain or if complications are suspected. Colonoscopy is contraindicated acutely and is only necessary for follow-up when age-appropriate cancer screening is indicated, or in cases of complicated disease. Antibiotics may not be necessary in all cases, and hospital admission is unnecessary for mild cases.

295
Q

Adverse effect of opioids on sex drive

A

Opioid use may cause numerous adverse reactions, including drowsiness, pruritus, nausea, and constipation. In addition to these well known side effects, chronic opioid use can lead to hypogonadism through inhibition of gonadotropin-releasing hormone and an increase in prolactin. A recently published study found that long-term opioid users were nearly twice as likely to be diagnosed with hypogonadism as short-term opioid users causing decreased libido. Given the large number of opioid users in the United States, prescribers should be aware of this adverse effect and screen for hypogonadism when appropriate.

296
Q

Next step in incidental ovarian cysts found on CT.

A

Ovarian incidentalomas are very common, and appropriate management depends upon the size and appearance of the incidentaloma as well as the menopausal status of the patient. The Society of Radiologists in Ultrasound states that simple cysts 5 cm or less in premenopausal women and simple cysts 3 cm or less in postmenopausal women are considered normal and do not require follow-up. The American College of Radiology recommends that immediate ultrasonography be performed in the evaluation of simple-appearing cysts that are incompletely characterized by CT and are >5 cm in premenopausal women or >3 cm in postmenopausal women. CA-125 levels have low sensitivity and specificity in premenopausal women and would not be indicated in this case.

297
Q

Sudden palpitations, chest tightness, lightheadedness in adult. Resolved with diltiazem. Symptoms last 10 minutes. What is condition and next step in evaluation.

A

This patient presents with a history consistent with typical atrioventricular nodal reentrant tachycardia, which is the most common type of supraventricular tachycardia (SVT). She is also using “pill-in-the-pocket” treatment, which is effective for infrequent SVT. Because the symptoms are episodic and the tachycardia is paroxysmal, patients generally present with normal examination and EKG findings. Further evaluation with event monitoring may identify a narrow-complex tachycardia with P waves hidden within the QRS complex or identified early after it. Most patients with SVT have structurally normal hearts.

298
Q

Differentiate episcleritis, conjunctivitis, iritis, and keratitis.

A

An acute red eye is a common presentation in primary care and it is critical to differentiate serious causes from benign causes. Episcleritis is a self-limited condition that can be idiopathic and presents with mild discomfort and focal hyperemia. Conjunctivitis is typically associated with a discharge that is clear in viral cases and mucopurulent in bacterial cases. Iritis is associated with significant pain, a poorly reactive pupil, diminished vision, and photophobia. This patient does not have changes in visual acuity, photophobia, or severe pain as seen in keratitis, which would also cause an abnormal fluorescein stain showing corneal ulceration.

299
Q

Next step in simple febrile seizure in child.

A

Children who have a simple febrile seizure and appear neurologically intact do not require routine testing except to determine the source of their fever (SOR C). This child has signs of possible pneumonia so a chest radiograph would be warranted to look for the source of infection that triggered the fever.
Routine laboratory testing is not indicated in the workup of simple febrile seizures. There is a low risk that these children will have low sodium or glucose levels, and this would not predict seizure recurrence. A lumbar puncture is indicated only in cases where the child has neurologic findings suggestive of meningitis, but that is not the case for this child.

300
Q

What developmental milestone is expected for a 12-month-old child

A

Standing independently is an expected developmental milestone for a 12-month-old child.

301
Q

Signs of temporomandibular disorder (TMD).

A

Approximately 10%–15% of adults will experience a temporomandibular disorder (TMD). TMD represents a spectrum of illnesses frequently seen and readily treatable by family physicians. This multifactorial disorder is consistently associated with other pain conditions such as fibromyalgia, as in this case. Patients typically present with facial pain, jaw pain, headache, or ear pain. The symptoms are generally associated with jaw movement such as chewing. Physical examination findings can be broad but often include pain with palpation of the temporomandibular joint, or pain and/or spasm of the muscles of mastication. Giant cell arteritis should be included in the differential diagnosis but is typically seen in patients over age 50 and is often associated with other findings such as jaw claudication, visual symptoms, and palpable abnormalities over the temporal artery. Sinusitis would usually present with nasal congestion, maxillary sinus tenderness, mucus, and fever. While trigeminal neuralgia may have a similar distribution of pain, this is typically reported as a brief attack of intense, sharp pain often produced by specific stimuli.

302
Q

Treatment for cough in young children.

A

Safe and effective options to treat cough in young children include nasal saline irrigation, a menthol rub, and honey (in children 12 months of age or older). Codeine should no longer be used for cough in anyone under 18 years of age. Over-the-counter cough and cold medications are not recommended for children under 4 years of age due to the lack of evidence of benefit and the significant side effects.

303
Q

How to bring about systolic murmur in hypertrophic cardiomyopathy?

A

Hypertrophic cardiomyopathy is the most common primary cardiomyopathy and is defined by left ventricular hypertrophy (primarily septal thickening) without cardiac dilation that may cause left ventricular outflow obstruction. It is a potential cause of sudden cardiac death. Many patients are asymptomatic and may potentially only be diagnosed by auscultation of a systolic murmur on examination that increases in intensity during Valsalva maneuvers (decreases LV filling).

304
Q

Who to check vitamin D levels?

A

The measurement of vitamin D levels is recommended only for patients with decreased kidney function, various skeletal diseases, or hypercalcemia (SOR C). Vitamin D deficiency is common in patients with chronic kidney disease, and it is associated with cardiovascular morbidity and mortality in those patients. However, good-quality evidence is lacking regarding the effect of vitamin D supplementation in these patients. There is no recommendation for screening the general population for vitamin D deficiency (SOR B). Vitamin D supplementation has not been found to improve depression, osteoarthritis, chronic pain, or fatigue (SOR A).

305
Q

Glucocorticoids use can cause what to bones?

A

Glucocorticoid use is the most common cause of secondary osteoporosis. Glucocorticoids increase bone resorption early, so addressing the issue at the start of treatment is vital in preventing fractures. The use of glucocorticoids is associated with an increased risk of fracture in the first 6 months.
A bone density test should be performed shortly after starting corticosteroid treatment (SOR C). The fracture risk can then be calculated using the FRAX assessment tool. If the risk of major osteoporotic fracture is sufficiently elevated, then treatment is recommended. One caveat is that the FRAX score should be adjusted upward if the prednisone dosage is >7.5 mg daily.
Calcium alone has not been shown to reduce the risk of fracture in osteoporosis. Calcium and vitamin D together have been shown to prevent decreases in bone mineral density with low-dose prednisone use but the effect with high doses is unknown. If a patient has an increased fracture risk, oral bisphosphonates should be started.

306
Q

rotator cuff muscle functions

A

Subscapularis tendon - Internal rotation of the shoulder. The inability to keep her hand away from her body when it is placed behind her back describes a positive internal lag test, also suggesting involvement of the subscapularis tendon.
Infraspinatus and teres minor - External rotation.
Supraspinatus and deltoid - Abduction of the shoulder.

307
Q

Which GLP-1 agonist lowers risk of recurrent cardiovascular events?

A

Liraglutide, exenatide, and dulaglutide are all GLP-1 receptor agonists. Only liraglutide has been shown to lower the risk of recurrent cardiovascular events and has received FDA approval for this indication. Empagliflozin, an SGLT2 inhibitor, has been associated with secondary prevention of cardiovascular disease.

308
Q

What is the content in oral rehydration solution for diarrhea in children?

A

Family physicians often see patients with diarrheal illnesses and most of these are viral. The World Health Organization recommends oral rehydration with low osmolarity drinks (oral rehydration solution) and early refeeding. Half-strength apple juice has been shown to be effective, and it approximates an oral rehydration solution. Its use prevents patient measurement errors and the purchase of beverages with an inappropriate osmolarity. Low osmolarity solutions contain glucose and water, which decrease stool frequency, emesis, and the need for intravenous fluids compared to higher osmolarity solutions like soda and most sports drinks. Water increases the risk of hyponatremia in children. Early refeeding has been shown to decrease the duration of illness.

309
Q

What are possible complications from subclinical hyperthyroidism?

A

Subclinical hyperthyroidism is evidenced by low TSH level with normal free T4 and free T3 levels. Common causes of subclinical hyperthyroidism include Graves disease, autonomous functioning thyroid adenoma, and multinodular toxic goiter. Subclinical hyperthyroidism may progress to overt hyperthyroidism; this is more likely in patients with TSH levels <0.1 μU/mL. Even in the absence of overt hyperthyroidism these patients are at higher risk for several health conditions, including atrial fibrillation, heart failure, and osteoporosis. It is important to assess for these conditions and consider treating the underlying thyroid condition, as well as the complication. The American Thyroid Association recommends treating patients with complications who are either over age 65 or have a TSH level <0.1 μU/mL.

310
Q

Findings of positive Dix-Hallpike maneuver in BPPV testing?

A

Benign paroxysmal positional vertigo (BPPV) originates in the posterior semicircular canal in the majority of patients (85%–95% range reported). The Dix-Hallpike maneuver, which involves moving the patient from an upright to a supine position with the head turned 45° to one side and the neck extended 20° with the affected ear down, will elicit a specific series of responses in these patients. Following a latency period that typically lasts 5–20 seconds but sometimes as long as 60 seconds, the patient will experience the onset of rotational vertigo. The objective finding of a torsional, upbeating nystagmus will be associated with the vertigo. The vertigo and nystagmus typically increase in intensity and then resolve within 1 minute from onset.

311
Q

Treatment for asthma in pediatrics.

A

Pediatric asthma is the most commonly encountered chronic illness, occurring in nearly one out of seven individuals. Short-acting β-agonists (albuterol) in the form of metered-dose inhalers are clearly favored for acute exacerbations, as well as for intermittent asthma. Treatment for persistent asthma requires the use of inhaled corticosteroids, with short-acting β-agonists used for exacerbations. For patients not well controlled with those options, either a long-acting β-agonist (salmeterol) or a leukotriene receptor antagonist (montelukast) may be added.

312
Q

American Society of Anesthesiologists (ASA). ASA II patient having major surgery (i.e total hip replacement) needs what for preop tests?

A

The American Society of Anesthesiologists (ASA) has recently revised its physical status classification system. A healthy patient would be classified as ASA I, while a patient with mild systemic disease would be classified as ASA II. All patients who are having major surgery should be offered preoperative laboratory testing, including a CBC and renal function testing. For patients who are ASA III or IV and have chronic liver disease or take anticoagulants, coagulation testing should be considered. There is no compelling evidence to support either a chest radiograph or an EKG as part of a routine preoperative evaluation.

313
Q

Treatment for trigger finger vs Dupuytren’s contracture?

A

Trigger finger - Flexion of finger due to tendon in finger is inflamed. Conservative treatment with activity modification, short course of NSAIDs, splinting finger, steroid injection. Surgery last resort.

Dupuytren’s contracture - fibroses nodule in palm, pulling finger towards palm (usually ring or 5th finger). Treatment is surgical release. Surgery is indicated when the metacarpophalangeal joint contracture reaches 30° or with any degree of contracture of the
proximal interphalangeal joint.

314
Q

Treatment for traveler’s diarrhea

A

Traveler’s diarrhea is the most common infection in international travelers. A short course of antibiotics
can be taken after a traveler develops diarrhea and usually shortens the duration of symptoms (SOR A).
Azithromycin is preferred to treat severe traveler’s diarrhea.

315
Q

What to order to confirm ovulation occurred in evaluating female infertility?

A

Female infertility. For women who are having regular menstrual cycles, ovulation is very likely. Ovulation can be confirmed by a progesterone level >3 ng/mL on day 21 of the cycle. If this is the case, tubal patency should be confirmed with hysterosalpingography or laparoscopy. Obstruction or adhesions would require surgical correction, but if there are none, referral for assisted reproductive technology would be appropriate.

Should the progesterone level be <3 ng/mL, anovulation should be investigated with TSH, estradiol, FSH, and prolactin levels.

316
Q

Treatment for bile acid diarrhea due to ileal resection?

A

Diarrhea that develops in patients with ileal Crohn’s disease or following ileal resection is usually due to increased amounts of bile acid remaining in the stool. This affects colonic secretion and motility and various protein factors in the gut, resulting in the development of bile acid diarrhea (BAD). A therapeutic trial with a bile acid sequestrant such as cholestyramine is most often used for both the diagnosis and treatment of BAD. Reducing fat intake may also be beneficial. Loperamide can lessen the diarrhea in some patients but should not be the primary treatment because chronic use can result in constipation. Fiber supplementation may help to produce a more formed stool and could be used as an adjunct treatment when appropriate.

317
Q

Which test is most accurate to detect anterior cruciate ligament tear?

A

The Lachman test is the most accurate test for an anterior cruciate ligament (ACL) tear (SOR A). The Lachman test has higher validity based on a sensitivity of 68% for partial ruptures and 96% for complete ruptures. The other two commonly used tests are the anterior drawer test, which has a sensitivity of 38% and a specificity of 81%, and the pivot shift test, which is more technically difficult than the other two tests and has a sensitivity ranging from 24% to 85%.

318
Q

Risks of long term PPI use

A

Risks of long-term use of PPIs have emerged. Currently known risks include increased fractures of the hip, wrist, and spine (SOR B), community-acquired pneumonia (SOR B), Clostridium difficile and other enteric infections (SOR C), hypomagnesemia (SOR B), and cardiac events when. coadministered with clopidogrel (SOR B). PPIs may also affect the absorption of vitamins and minerals,
including iron, vitamin B12, and folate (SOR C).

319
Q

Treatment for Immune (idiopathic) thrombocytopenic purpura.

A
Immune (idiopathic) thrombocytopenic purpura is an acquired immune-mediated disorder defined as
isolated thrombocytopenia not found to have another cause. Treatment is usually restricted to severe
thrombocytopenic cases (platelet count <50,000/mm3) unless there is evidence of acute bleeding. Corticosteroids are considered the first-line therapy (SOR C). Intravenous immunoglobulin and rituximab have also been used as first-line agents. Second-line therapies include thrombopoietin-receptor agonists and splenectomy. Further evaluation, including a bone marrow biopsy, to rule out myelodysplastic syndrome and lymphoproliferative disorders is indicated in patients over the age of 60 (SOR C). Platelet transfusion is not indicated in the absence of hemorrhage or a need for surgery.
320
Q

Treatment for blepharitis.

A

Blepharitis is a chronic inflammation of the eyelids. Seborrhea is a common cause in older adults. In younger patients including children, colonization with Staphylococcus may be a contributing
factor. Meibomian gland dysfunction is often part of this condition, contributing to a reduced quality of
tear films, which leads to dry eyes and irritation.
Initial treatment of blepharitis is lid hygiene using warm compresses to remove dried secretions and debris. Mild shampoo can help in this process and aid in keeping the bacterial colonization load down. In severe or recalcitrant cases a topical antibiotic ointment may be applied to the lids. Oral antibiotics can be considered for more severe cases.

321
Q
A

Trichomoniasis classically presents as a greenish-yellow, frothy discharge with a foul odor. Erythema and inflammation of the vagina and cervix are often present and can include punctate hemorrhages (strawberrycervix). Bacterial vaginosis more commonly presents as a thin, homogenous discharge with a fishy odor and no cervical or vaginal inflammation. Vulvovaginal candidiasis presents with white, thick, cheesy, or curdy discharge. Atrophic vaginitis may cause a thin, clear discharge and is usually associated with a thin, friable vaginal mucosa. Irritant/allergic vaginitis causes burning and soreness with vulvar erythema but usually does not cause any significant discharge.

322
Q

Goal fasting BG in gestational diabetes, 1 hour postprandial, and 2 hour postprandial?

A

The goal fasting blood glucose level in patients with gestational diabetes is <95 mg/dL. A fasting glucose
level <80 mg/dL is associated with increased maternal and fetal complications. The goal 2-hour postprandial glucose level is <120 mg/dL and the goal 1-hour postprandial glucose level is <140 mg/dL.

323
Q

Beneficial treatments for IBS symptoms.

A

Irritable bowel syndrome (IBS) symptoms improve with several different medications and alternative therapies. Exercise, probiotics, antibiotics, antispasmodics, antidepressants, psychological treatments, and peppermint oil all have evidence that they may improve IBS symptoms (SOR B). A Cochrane review of 15 studies involving 922 patients found a beneficial effect from antidepressants with regard to improvement in pain and overall symptom scores compared to placebo. SSRIs used in these trials included citalopram, fluoxetine, and paroxetine, and tricyclic antidepressants included amitriptyline, desipramine, and imipramine.

324
Q

What is most sensitive and specific method of diagnosis of genital herpes in pregnancy?

A

When genital herpes occurs during pregnancy, the best method of diagnosis is either a tissue culture or a
polymerase chain reaction (PCR) test, which is more sensitive.

325
Q

Treatment of Takotsubo cardiomyopathy.

A

Takotsubo cardiomyopathy, also known as stress-induced cardiomyopathy, can develop following
emotional distress and is characterized by the abrupt onset of dysfunction of the left ventricle. The clinical
presentation and laboratory studies can mirror acute coronary syndrome and should be treated similarly.
Once symptoms and cardiac abnormalities resolve, treatment is no longer indicated and may be withdrawn
if there are no signs of coronary disease. Because this patient currently has cardiomyopathic abnormalities,
a diuretic, ACE inhibitor, and -blocker are indicated.

326
Q

Which tests predicts severity of pancreatitis?

A

Knowing the severity of pancreatitis helps predict how aggressive management should be. Hematocrit,
BUN, and creatinine levels are the most useful predictors of the severity of pancreatitis, reflecting the degree of intravascular volume depletion. C-reactive protein is often elevated, but it is not as useful as hematocrit for predicting severity. Serum amylase and lipase have no prognostic value. CT evidence of severe pancreatitis lags behind clinical and laboratory evidence, and early CT underestimates the severity of the acute process.

327
Q

Treatment for De Quervain’s tenosynovitis

A

De Quervain’s tenosynovitis is a painful condition affecting the tendons on the thumb side of the wrist.
Exact etiology unknown. It usually occurs with repeated use of the thumb and is characterized by pain in the radial wrist. The course is typically self-limited but can last for up to a year. Conservative therapy with immobilization and NSAIDs is recommended if there are no contraindications to NSAIDs. A corticosteroid injection is helpful but is typically reserved for severe cases or if conservative therapy fails. Surgery may be beneficial but is generally not recommended unless the course is severe, given the natural history of resolution.

328
Q

Topiramate increase risk of what?

A

Topiramate increases the risk of kidney stones. It is a carbonic anhydrase inhibitor, which induces a
metabolic acidosis that leads to hypercalciuria and the formation of calcium phosphate stones.

329
Q

Treatment for fracture of the fifth metatarsal in foot

A

The fifth metatarsal has the least cortical thickness of all of the metatarsals. There are strong ligaments and
capsular attachments on the proximal fifth metatarsal that can put significant stress on this area of the bone, leading to fractures. Nondisplaced tuberosity fractures can generally be treated with compressive dressings such as an Aircast or Ace bandage, with weight bearing and range-of-motion exercises as tolerated. Minimally displaced (<3 mm) avulsion fractures of the fifth metatarsal tuberosity can be treated with a short leg walking boot. If the displacement is >3 mm, an orthopedic referral is warranted.

330
Q

Should you order imaging for uncomplicated acute low back pain and/or radiculopathy?

A

Uncomplicated acute low back pain and/or radiculopathy is a benign, self-limited condition and early imaging is associated with worse overall outcomes and is likely to identify minor abnormalities even in asymptomatic patients. Imaging for acute low back pain should be reserved for cases that are suspicious for cauda equina syndrome, malignancy, fracture, or infection. In the absence of red flags such as progressive motor or sensory loss, new urinary retention or overflow incontinence, a history of cancer, a recent invasive spinal procedure, or significant trauma relative to age, imaging is not warranted regardless of whether radiculopathy is present, unless symptoms persist despite a trial of at least 6 weeks of medical management and physical therapy.

331
Q

Treatment for vasomotor symptoms of perimenopause?

A

This patient presents with typical vasomotor symptoms that can begin in perimenopause and affect sleep
quality. Hormone therapy is the gold standard for treatment of vasomotor symptoms. Combination estrogen and progesterone therapy is highly effective for vasomotor symptoms and provides protection against uterine neoplasia

332
Q

Treatment for symptoms Wolff-Parkinson-White syndrome?

A

Catheter ablation is the most appropriate treatment for a patient with symptomatic Wolff-Parkinson-White
syndrome (WPW). Catheter ablation has a very high immediate success rate (96%–98%). The most
significant risk associated with the procedure is permanent atrioventricular block, which occurs in
approximately 0.4% of procedures.

333
Q

CDC recommendation for influenza vaccine in previously unvaccinated patients ages 6 months to 8 years old.

A

The CDC’s Advisory Committee on Immunization Practices recommends that patients with egg allergy
receive influenza vaccination. Previously unvaccinated patients ages 6 months to 8 years should receive
two doses of either trivalent or quadrivalent vaccine separated by 1 month.

334
Q

Hip physical exam site of pathology?
Positive FABER test
Positive log roll test
Postive FADIR test

A

The cause of hip pain is generally determined from the patient’s history and physical examination. A
positive flexion, abduction, external rotation (FABER) test that produces pain at the sacroiliac joint, lumbar
spine, and posterior hip is associated with sacroiliac joint dysfunction. The log roll test involves passive
supine internal and external rotation of the hip. When this test is positive for pain it is associated with
piriformis syndrome. While femoroacetabular impingement may be associated with a positive FABER test, it would produce pain in the groin. Greater trochanteric pain syndrome results in lateral hip pain rather than posterior pain. Osteoarthritis is usually associated with a limited range of motion and groin pain.

Think low back, posterior hip site pain on positive FABER is sacroiliac joint pathology.
Think groin pain on positive FABER is hip joint (femoroacetabular joint) pathology.

Think groin pain on positive FADIR test is hip joint (femoroacetabular joint) pathology.

335
Q

treatment for primary focal hyperhidrosis

A

The recommended first-line treatment for primary focal hyperhidrosis is topical 20% aluminum chloride.
It should be applied to affected areas nightly for 6–8 hours and works by obstructing the eccrine sweat
glands and destroying secretory cells. Topical 2% glycopyrrolate must be compounded by a pharmacy and is indicated only for craniofacial hyperhidrosis.

336
Q

Treatment for acute epididymitis.

A

This patient presents with acute epididymitis. Typical symptoms develop gradually over 1–2 days with
posterior scrotal pain and swelling. Additional symptoms may include fever, hematuria, dysuria, and
urinary frequency. The pain may radiate to the lower abdomen. Physical examination findings may include
tenderness of the epididymis and testis along with swelling of the scrotum. Elevation of the scrotum may
decrease the pain (Prehn sign). Typical ultrasound findings include hyperemia, swelling, and increased
blood flow to the epididymis.
With testicular torsion the pain is often sudden in onset and severe, with associated nausea and vomiting and no other urologic symptoms. A physical examination often demonstrates a high-riding testis that may lie transversely in the scrotum. The cremasteric reflex may be absent. Ultrasound findings would demonstrate decreased or absent blood flow with testicular torsion.
In sexually active adults <35 years of age, gonorrhea and Chlamydia are the most common causative
organisms of acute epididymitis. Ceftriaxone, 250 mg intramuscularly or intravenously once, with oral
doxycycline, 100 mg twice daily for 10 days, would be the appropriate treatment for acute epididymitis
(SOR C). In men over the age of 35 or those with a history of recent urinary tract surgery or
instrumentation, enteric organisms are the most likely cause and monotherapy with oral levofloxacin or
ofloxacin for 10 days would be the recommended treatment.

337
Q

The normal TSH range in pregnancy is lower or higher than in the nonpregnant state.

A

The TSH reference range is lower during pregnancy because of the cross-reactivity of the -subunit of
hCG. Levels of hCG peak during weeks 7–13 of pregnancy, and hCG has mild TSH-like activity, leading
to slightly high free T4 levels in early pregnancy. This leads to a feedback decrease in TSH.

338
Q

What are the signs of multiple myeloma and what to order to diagnose?

A
Multiple myeloma (MM) is neoplastic proliferation of plasma cells producing a monoclonal immunoglobulin. 
MM signs include bone pain from osteolytic lesions, normocytic anemia, renal insufficiency. The laboratory findings along with worsening back pain indicate a need to order serum protein electrophoresis to look for MM. Flow cytometry is generally used in patients with an elevated WBC count and suspected lymphoma.
339
Q

What medication is most likely to result in improved outcomes in severe community acquired pneumonia.

A

Corticosteroids such as methylprednisolone have been shown to improve clinical outcomes such as length of stay, duration of antibiotic treatment, and the risk of developing adult respiratory distress syndrome in severe community-acquired pneumonia. The preferred choice of antibiotic treatment for patients in the intensive-care unit is a beta-lactam antibiotic (ceftriaxone, cefotaxime) or ampicillin/sulbactam, plus a macrolide (azithromycin) alone or a macrolide and a respiratory fluoroquinolone.

340
Q

What is acute poststreptococcal glomerulonephritis (APSGN) and what findings on microscopic evaluation of the urine sediment would help to confirm the diagnosis in this patient?

A

This is a classic presentation for acute poststreptococcal glomerulonephritis (APSGN), with the onset of gross hematuria associated with hypertension and systemic edema. This is most commonly seen in school-age children, usually 1–2 weeks after an episode of pharyngitis or 3–4 weeks after an episode of impetigo, caused by so-called nephritogenic strains of Group A -hemolytic Streptococcus. The hematuria is caused by immune complex–mediated glomerular injury.

The classic finding on microscopic
urinalysis for acute glomerulonephritis is the presence of RBC casts. WBC casts are seen with acute pyelonephritis. The presence of urinary eosinophils indicates acute interstitial nephritis. Calcium oxalate makes up the most common type of kidney stones.
Antibiotics prescribed for antecedent pharyngitis do not prevent APSGN. Treatment is supportive,
controlling blood pressure and edema with a thiazide or a loop diuretic. The prognosis for resolution and
full recovery of the vast majority of patients with APSGN is excellent, especially in the pediatric age
group.

341
Q

What medication can be given in outpatient clinic for acute migraine?

A

Multiple studies have determined that parenteral antiemetics have benefits for the treatment of acute
migraine beyond their effect on nausea. Most outpatient clinics do not have the ability to administer
intravenous metoclopramide, which is the preferred treatment. However, most clinics do have the ability
to administer intramuscular prochlorperazine or promethazine. Oral butalbital/acetaminophen/caffeine and oral ergotamine/caffeine have less evidence of
success in the treatment of acute migraine. Sumatriptan is contraindicated within 24 hours of the use of rizatriptan.

342
Q

What are the clinical signs and imaging to diagnose calcaneal stress fracture?

A

This patient has a calcaneal stress fracture as suggested by the history of increased running on a hard surface, improvement with rest, and a positive calcaneal squeeze on examination. A delay in diagnosis increases the risk of delayed union. MRI is the preferred imaging modality because radiographs often do not detect a calcaneal stress fracture.

343
Q

What criteria to diagnosis POCS?

A

Several professional organizations have published criteria for the diagnosis of polycystic ovary syndrome
(PCOS) using various combinations of hyperandrogenism (clinical or biochemical), ovulatory dysfunction (typically oligomenorrhea), and the presence of at least one polycystic ovary by imaging criteria.
The National Institutes of Health advises that patients must have both hyperandrogenism and
oligomenorrhea to meet the criteria for PCOS. The Endocrine Society recommends diagnosing PCOS
based on the 2003 Rotterdam criteria, which require the presence of two of the following: hyperandrogenism, ovulatory dysfunction, and at least one polycystic ovary.
Hypertension, hypothyroidism, infertility, and obesity are common symptoms in patients with PCOS but
are not diagnostic.

344
Q

Clinical findings and treatment of eosinophilic esophagitis (EoE).

A

The clinical presentation and EGD findings indicate eosinophilic esophagitis (EoE) in this patient (biopsy reveals >15 eosinophils/hpf, Helicobacter pylori testing is negative, upper abdominal pain, no acid reflux). In the absence of other causes of eosinophilia, the presence of >15 eosinophils/hpf is considered diagnostic. Application of corticosteroids to the esophagus is generally the treatment of choice,
either in the form of an oral suspension of budesonide or an inhaled corticosteroid sprayed into the mouth and swallowed. EoE does not respond to proton pump inhibitor therapy or H2-blocker therapy and systemic corticosteroids are not necessary.

345
Q

Medications associated with osteoporosis and increased fracture risk include?

A

Medications reported to be associated with osteoporosis and increased fracture risk include antiepileptic drugs, long-term heparin, cyclosporine, tacrolimus, aromatase inhibitors, glucocorticoids,
gonadotropin-releasing hormone agonists, thiazolidinediones, excessive doses of levothyroxine, PPIs, SSRIs, parenteral nutrients, medroxyprogesterone contraceptives, methotrexate, andaluminum antacids.

346
Q

Characteristics of malignant vs benign lung nodules?

A

Characteristics that are more commonly associated with malignant lesions include a nonsolid “ground glass” appearance, a size >6 mm, noncalcified lesions, a lesion size or volume doubling time between 1 month and 1 year, and irregular or spiculated borders. Findings on a chest radiograph that are more commonly associated with benign lesions include a lesion size <6 mm, concentric or “popcorn-like” calcifications, doubling times of <1 month or >2–4 years, dense solid-appearing lesions, and lesions with smooth regular borders.

347
Q

Treatment for insomnia with behavioral interventions, Doxepin, and melatonin fail.

A

Although behavioral interventions are the mainstay of treatment for insomnia, they often need to be supplemented by pharmacologic therapy. When both doxepin and extended-release melatonin fail to provide benefit, a member of the Z-drug class should be tried next. Among the Z-drugs only eszopiclone (Lunesta) provides an early peak onset and a long half-life, with a 1-hour approximate time to peak and a 6-hour half-life. While zaleplon (Sonata) has an equally short time to peak of 1 hour, it also has a 1 hour half-life. Antihistamines, including diphenhydramine and doxylamine, as well as atypical antipsychotics such as olanzapine, are not indicated unless used primarily to treat another condition.

348
Q

Describe the types of headaches and treatment.
migraine
tension headache
cluster headaches
- chronic paroxysmal hemicrania (variant of cluster headache)

A

There are several types of chronic headaches, and they often respond to different treatments. Migraine is very prevalent and is characterized by headaches that are periodic, often unilateral, and frequently pulsatile. Migraine is familial and typically starts in childhood, adolescence, or young adulthood, and the headaches decrease in frequency over time. Some are associated with aura, which causes visual disturbances. In mild cases, over-the-counter medications may control symptoms. For most patients, however, treatment to control the attack can include triptans such as sumatriptan, and/or ergot alkaloids such as ergotamine. Treatment to prevent attacks may also be appropriate, and could include a β-blocker, antiepileptic drugs, or amitriptyline (TCA).

Tension headaches are usually bilateral and are typically described as dull or aching, but patients often describe tightness or pressure. They are not associated with symptoms such as throbbing, nausea, or photophobia. Tension headaches are more frequent than migraine but patients often treat them at home without seeking medical treatment. Frequent or persistent tension headaches can be treated with several drugs used for anxiety or depression, including amitriptyline. Stronger analgesics and ergotamine are not helpful.

Cluster headache is another type of chronic headache. This occurs most frequently in adult males, and often occurs over a period which may extend over many weeks, with repeated episodes or clusters. It most often occurs at night, and may recur several times during the night. The headache is unilateral and is associated with orbital pain and vasomotor phenomenon such as blocked nasal passages, rhinorrhea, conjunctival injection, and miosis. The headache can be treated with inhalation of 100% oxygen, and the headache cycle can be terminated with verapamil. Ergotamine or sumatriptan can be used at night to prevent attacks.

There are also variants of cluster headaches, including chronic paroxysmal hemicrania, which resembles cluster headache but has some important differences. Like cluster headaches, these headaches are unilateral and accompanied by conjunctival hyperemia and rhinorrhea. However, these headaches are more frequent in women, and the paroxysms occur many times each day. This type of headache falls into a group of headaches that have been labeled indomethacin-responsive headaches because they respond dramatically to indomethacin.

349
Q

What lung conditions have normal spirometry and low carbon monoxide diffusing capacity (DLCO)?

A

Low diffusing capacity of the lungs for carbon monoxide (DLCO) with normal spirometry indicates a disease process that disrupts gas transfer in the lungs without causing lung restriction or airflow obstruction. Common causes include chronic pulmonary emboli, heart failure, connective tissue disease with pulmonary involvement, and primary pulmonary hypertension. Asthma, bronchiectasis, COPD, and pulmonary fibrosis are associated with abnormalities on spirometry.

350
Q

Antibiotic prophylaxis prior to dental procedures for patients with prosthetic heart valve who have penicillin allergy.

A

Patients with an artificial heart valve should be given antibiotic prophylaxis prior to dental procedures to prevent infectious endocarditis. The organisms that most frequently cause infectious endocarditis include Staphylococcus aureus (31%), Streptococcus viridans (17%), coagulase-negative staphylococci (11%), Enterococcus (11%), Streptococcus bovis (7%), and other streptococci (5%). Amoxicillin is the preferred medication for prophylaxis. Clindamycin or azithromycin can be used in patients with a penicillin allergy. If the penicillin allergy is not associated with anaphylaxis, angioedema, or urticaria, then cephalexin would be an appropriate antibiotic choice.

351
Q

Examples of primary prevention, secondary prevention, and tertiary prevention.

A

Prevention traditionally has been divided into three categories: primary, secondary, and tertiary. Primary prevention targets individuals who may be at risk to develop a medical condition and intervenes to prevent the onset of that condition. Examples include childhood vaccination programs, water fluoridation, antismoking programs, and education about safe sex. Secondary prevention targets individuals who have developed an asymptomatic disease and institutes treatment to prevent complications. Examples include routine Papanicolaou tests and screening for hypertension, diabetes mellitus, or hyperlipidemia. Tertiary prevention targets individuals with a known disease, with the goal of limiting or preventing future complications. Examples include screening patients with diabetes for microalbuminuria, rigorous treatment of diabetes mellitus, and post–myocardial infarction prophylaxis with β-blockers and aspirin.

352
Q

Treatment for plaque psoriasis if patient fails topical steroids.

A

Oral corticosteroids are not indicated in the treatment of plaque psoriasis. All of the other options are indicated only if topical treatments fail. Of the options listed, the combination of a topical corticosteroid and topical calcipotriene is considered the most appropriate for this patient. Another option would be to add topical tazarotene to the topical corticosteroid. However, when tazarotene is used as monotherapy it often fails to clear plaques and increases the incidence of skin irritation.

353
Q

Treatment for plaque psoriasis if patient fails topical steroids.

A

Oral corticosteroids are not indicated in the treatment of plaque psoriasis. All of the other options are indicated only if topical treatments fail. Of the options listed, the combination of a topical corticosteroid and topical calcipotriene is considered the most appropriate for this patient. Another option would be to add topical tazarotene to the topical corticosteroid. However, when tazarotene is used as monotherapy it often fails to clear plaques and increases the incidence of skin irritation.

354
Q

Define age related macular degeneration

A

This patient presents with signs and symptoms that suggest age-related macular degeneration (central vision). Has positive Amsler grid testing with distorted lines. Smoking is a modifiable risk factor and smokers should be counseled to quit (SOR C). The patient should be referred to an ophthalmologist for further evaluation and management. Vitamin supplements with Age-Related Eye Disease (AREDS) and AREDS2 formulations have been shown to delay visual loss in patients with age-related macular degeneration (SOR A). Age-related macular degeneration is not reversible but treatment can delay progression or stabilize the changes (SOR A).

355
Q

Treatment for COPD based on GOLD category A, B, C

A

This patient has COPD and is in a risk category of A (low risk, fewer symptoms) based on the Global Initiative for Chronic Obstructive Lung Disease (GOLD) combined assessment of COPD. As a result, either a short-acting anticholinergic or a short-acting β2-agonist should be selected as the initial pharmacologic management. Long-acting β2-agonists or long-acting anticholinergics are indicated for patients with a GOLD combined assessment category of B or worse. Long-acting inhaled corticosteroids are indicated for patients with a GOLD combined assessment category of C or worse.

356
Q

What test to use for acute mental status changes (delirium) in geriatrics?

A

The patient is experiencing an acute cognitive change from baseline, indicating possible delirium. The Confusion Assessment Method (CAM) is a delirium diagnosis tool useful for evaluating acute cognitive changes. The other tests listed, including the Mini-Mental State Examination, Mini-Cog, Montreal Cognitive Assessment, and Saint Louis Mental Status exam, test chronic baseline cognitive function and are not designed to test for acute changes.

357
Q

SSRIs improve what symptom in fibromyalgia

A

Fibromyalgia is a chronic complex condition characterized by muscle pain, fatigue, muscle tenderness, and sleep disorders, often accompanied by mood disorders. SSRIs have been studied in the treatment of these symptoms, and while they have been shown to produce up to a 30% reduction in pain scores in patients with fibromyalgia, they have not been shown to affect fatigue or sleeping problems. They also have not been shown to be superior to TCAs (amitriptyline) when treating pain. As with other patient populations, SSRIs have been shown to improve depression in those with fibromyalgia.

358
Q

PFT criteria for COPD diagnosis.

A

COPD is present if the FEV1/FVC ratio is reduced to <70% and is irreversible with bronchodilator therapy.

359
Q

Criteria for diagnosing ADHD.

A

Diagnosing attention-deficit disorder in adults requires symptoms that interfere with social, academic, or occupational functioning and are present in more than one setting. DSM-5 states that a history of symptoms before age 12 is required for the diagnosis.

360
Q

What medication to prevent pregnancy and improve hirsutism in PCOS.

A

In this patient with PCOS who needs contraception and hopes to address her hirsutism, combined oral contraceptives are most likely to address both concerns. In addition to suppressing ovulation they also suppress gonadotropin and ovarian androgen production. The estrogen component increases hepatic production of sex hormone binding globulin, thus decreasing androgen bioavailability.

Progestin-only pills and the levonorgestrel IUD protect against pregnancy but will not improve hirsutism.

361
Q

Which coexisting conditions could require a higher dose of levothyroxine (Synthroid) to adequately treat hypothyroidism?

A

Absorption of levothyroxine is impaired by several gastrointestinal conditions, including atrophic gastritis, chronic proton pump inhibitor use, and Helicobacter pylori infection. Treatment of H. pylori infection reverses this effect, and following eradication of the infection a reduction of the levothyroxine dosage by 30% or more will often be required.

362
Q

The diameter of which heart chamber best predicts the likelihood that sinus rhythm will be maintained after successful cardioversion from atrial fibrillation?

A

If atrial fibrillation is converted back to sinus rhythm, the likelihood of the patient staying in sinus rhythm is best predicted from the diameter of the left atrium on the patient’s echocardiogram. Significant left atrium enlargement means the patient is unlikely to stay in sinus rhythm after successful conversion.

Other factors that predict a lack of success in maintaining sinus rhythm after cardioversion include a longer time in atrial fibrillation before cardioversion, or the presence of underlying heart disease, especially rheumatic heart disease.

363
Q

Treatment of the two subtypes of functional dyspepsia

  • epigastric pain syndrome
  • postprandial distress syndrome
A

This patient’s history fits the diagnosis of functional dyspepsia. Two subtypes of this disorder have been described. The first, epigastric pain syndrome, is described as intermittent pain and burning in the epigastrium. The second, postprandial distress syndrome, is more typical of the symptoms this patient describes: postprandial fullness and early satiety. Although there is considerable benefit from reassurance and “naming” a patient’s condition, empiric treatment is also warranted. Patients with epigastric pain syndrome are more likely to respond to proton pump inhibitors or H2-blockers. Patients with predominantly postprandial distress symptoms are more likely to improve with a motility agent such as metoclopramide.

364
Q

Define positive predictive value, negative predictive value, and likelihood ratio.

A

There has been a large increase in the number of diagnostic tests available over the past 20 years. Although tests may aid in supporting or excluding a diagnosis, they are associated with expense and the potential for harm. In addition, the characteristics of a particular test and how the results will affect management and outcomes must be considered. Clinically useful statistics for evaluating diagnostic tests include the positive predictive value, negative predictive value, and likelihood ratio.

The likelihood ratio indicates how a positive or negative test correlates with the likelihood of disease. Ratios greater than 5–10 greatly increase the likelihood of disease, and those less than 0.1–0.2 greatly decrease it. In the example given, if the patient’s endometrial stripe is >25 mm, the likelihood ratio is 15.2 and her post-test probability of endometrial cancer is 63%. However, if it is ≤4 mm, the likelihood ratio is 0.02 and her post-test probability of endometrial cancer is 0.2%.

The number needed to treat is useful for evaluating data regarding treatments, not diagnosis. Prevalence is the existence of a disease in the current population, and incidence describes the occurrence of new cases of disease in a population over a defined time period. The relative risk is the risk of an event in the experimental group versus the control group in a clinical trial.

365
Q

(USPSTF) diabetes screening recommendation

A

The U.S. Preventive Services Task Force (USPSTF) recommends screening for abnormal blood glucose levels as part of a cardiovascular disease risk assessment for adults who are 40–70 years of age and who are overweight (BMI 25.0–29.9 kg/m²) or obese (BMI ≥30.0 kg/m²). Since his fasting blood glucose result was in the range consistent with impaired fasting glucose (100–125 mg/dL), the USPSTF recommends confirming the diagnosis of the abnormal glucose level soon by performing the same test on a different day. Appropriate treatment should begin once the diagnosis is confirmed.

366
Q

Treatment for infantile hemangiomas and hemangiomas that have involuted?

A

Infantile hemangiomas usually appear by 4 weeks of age and stop growing by 5 months of age. As many as 70% leave residual skin changes, including telangiectasia, fibrofatty tissue, redundant skin, atrophy, dyspigmentation, and scarring. Systemic corticosteroids were the mainstay of treatment for hemangiomas during infancy until 2008, when the FDA approved oral propranolol for this indication. Intralesional corticosteroids can be effective for small, bulky, well localized lesions in infants. Laser therapy can also be used to treat early lesions or residual telangiectasia. Once involution is complete, however, elective surgical excision is the treatment of choice, producing better outcomes.

367
Q

What is the recommended timing for long-acting reversible contraception (LARC) placement?

A

Long-acting reversible contraception (LARC) includes the copper IUD, levonorgestrel IUDs, and subdermal implants. LARCs can be placed at any point in the patient’s menstrual cycle (SOR A). There should be evidence that the patient is not pregnant prior to placement.

368
Q

Pleural fluid criteria for transudative vs exudative fluid and examples?

A

Evaluation of the pleural fluid is important to assist in determining the cause of the effusion. Protein, glucose, LDH, and cell counts should be measured in the pleural fluid. This can help determine if the effusion is exudative or transudative. A pleural protein to serum protein ratio >0.5 or a pleural fluid LDH to serum LDH ratio >0.6 suggests an exudative effusion. Lower ratios suggest a transudative process. With transudates, the pleural fluid pH is typically between 7.40 and 7.55, with fewer than 1000 WBCs, and the glucose level is similar to the serum glucose level. Cirrhosis with ascites is a cause of transudative effusion. Pleural effusions associated with malignancy, pneumonia, viral illness, and asbestosis tend to be exudative.

369
Q

What is a cause of pseudohyponatremia?

A

Plasma sodium concentration measurements can be unreliable in patients with severe hyperlipidemia or hyperproteinemia (pseudohyponatremia).

370
Q

Define categories of asthma and treatment for it.

Intermittent, mild persistent, moderate persistent, and severe asthma

A

In patients ≥12 years of age, asthma is classified as intermittent if symptoms are present ≤2 days per week, nighttime awakenings occur ≤2 times per month, an inhaler is required ≤2 days per week, and the FEV1 is >80% of predicted. Intermittent asthma does not interfere with normal activities and inhaled short-acting β2-agonists during symptomatic periods are usually sufficient treatment.
Mild persistent asthma is defined as symptoms present >2 days per week but not daily, nighttime awakenings 3–4 times per month, and inhaler use >2 days per week but not daily and not more than once on any day. The FEV1 is >80% of predicted. Mild persistent asthma can cause minor limitations during normal activities and should be treated with low-dose inhaled corticosteroids (ICs).
With moderate persistent asthma, symptoms are present daily, nighttime awakenings occur >1 time per week but not nightly, and an inhaler is required daily. A patient with moderate persistent asthma has an FEV1 that is 60%–80% of predicted and can experience some limitations during normal activities. Moderate persistent asthma is treated with a combination of low-dose ICs and long-acting β2-agonists (LABA) or medium-dose ICs as monotherapy.
Severe asthma is defined as symptoms present throughout the day, nighttime awakenings up to 7 times per week, inhaler use several times per day, and an FEV1 <60% of predicted. Normal activities are extremely limited by severe asthma. Treatment includes medium- to high-dose ICs with a LABA, and consultation with an asthma specialist is recommended. Omalizumab is also indicated for patients who have allergies.

371
Q

What is an adverse effect of ranitidine?

A

Thrombocytopenia is a relatively common dyscrasia often discovered through routine laboratory studies. There are many causes of thrombocytopenia but medication-induced thrombocytopenia should always be considered. In this case the patient is taking an H2-blocker that may cause blood dyscrasias. The offending agent should be stopped and a repeat level should be obtained in 2–4 weeks for patients with mild asymptomatic thrombocytopenia (platelet count 100,000–150,000/mm3) and in 1–2 weeks for moderate thrombocytopenia (platelet count 50,000–100,000/mm3) (SOR C). Prednisone is the
first-line treatment for immune thrombocytopenic purpura (SOR C); however in this situation, it is reasonable to look for other causes first. If the thrombocytopenia is worse on repeat testing (<100,000/mm3) it is reasonable to consider consultation for further evaluation and to consider a bone marrow biopsy. If the platelet count improves, continued monitoring is indicated until it returns to normal. Platelet transfusions are not indicated in stable, nonbleeding patients unless the platelet count drops below 10,000/mm3.

372
Q

Rises in how much serum creatinine is tolerable when starting ACEI or ARBs?

A

It is recommended that physicians should tolerate a rise of <30% in serum creatinine after ACE inhibitor or angiotensin receptor blocker (ARB) initiation. Rises in serum creatinine of >30% from baseline increase the risk of renal failure, adverse cardiac outcomes, and death. A recent study suggests that rises in serum creatinine of <30% also put patients at risk for these outcomes, with a dose-response relationship between the magnitude of creatinine change and the risk of adverse outcomes.

This patient has more than a 30% rise in creatinine and has no other factors, such as diabetes mellitus, heart failure, or chronic kidney disease, that would indicate a need for ACE or ARB therapy for her hypertension. Discontinuing her ACE inhibitor and starting a medication from a different class is the most appropriate treatment at this time. Based on JNC 8 guidelines, additional options for blood pressure medications include thiazide diuretics and calcium channel blockers.

373
Q

Intensive behavioral intervention has more benefit than other treatment modalities in treating children who have been diagnosed with

A

The only evidence-based treatment that confers significant benefits to children with autism is intensive behavioral interventions, which should be initiated before 3 years of age.

374
Q

What is next step in a suspected distal biceps tendon rupture?

A

In a patient with a suspected distal biceps tendon rupture, clinical signs can be unreliable and MRI imaging is the test of choice. A Speed’s test is used to evaluate pain related to the long head of the biceps tendon. Surgical repair is the treatment of choice when the tendon is ruptured. Physical therapy and local corticosteroid injections are not beneficial.

375
Q

Peak effect of opioids when given PO, SQ/IM, and IV?

A

Most orally administered immediate-release opioids such as morphine, oxycodone, and hydromorphone reach their peak effect at about 1 hour, at which time additional medication can be given if the patient is still in pain. Intravenous opioids reach their peak effect at about 10 minutes and intramuscular and subcutaneous opioids at about 20–30 minutes. Additional medication may therefore be given at those intervals if additional pain relief is required.

376
Q

What would be most effective for decreasing mortality in pulmonary hypertension related to COPD?

A

The only proven therapy for pulmonary hypertension related to COPD is supplemental oxygen. Supplemental oxygen should be recommended when the PaO2 is <60 mm Hg, because it has been shown to improve mortality by lowering pulmonary arterial pressures. Treatments effective for pulmonary artery hypertension should not be used. Pulmonary vasodilators such as nifedipine, sildenafil, and bosentan may cause a ventilation-perfusion mismatch. Pulmonary endarterectomy may be indicated for pulmonary hypertension caused by chronic thromboembolic disease.

377
Q

Which antihypertensive drugs may reduce the severity of sleep apnea?

A

Diuretics lessen the severity of obstructive sleep apnea and reduce blood pressure. Aldosterone antagonists offer further benefit beyond that of traditional diuretics. Resistant hypertension is common in patients with obstructive sleep apnea. Resistant hypertension is associated with higher levels of aldosterone, which can lead to secondary pharyngeal edema, increasing upper airway obstruction.

378
Q

What is IVF rate for sepsis?

A

The Surviving Sepsis Campaign recommends that patients with elevated serum lactate or hypotension receive isotonic intravenous fluids such as NS or lactated Ringer’s solution at an initial rate of 30 mL/kg in the first 3 hours using small boluses of approximately 500 mL.

379
Q

A past history of which one of the following would indicate the need for MRI of the breasts?

A

MRI of the breasts should be reserved for women at very high risk for breast cancer such as those with genetic mutations, a history of breast irradiation, or a very high-risk family history. Women who had chest radiation therapy during childhood or adolescence, generally for Hodgkin’s disease, are at an extremely high risk for breast cancer.

380
Q

Treatment for Bell’s palsy.

A

Treatment for Bell’s palsy (idiopathic facial paralysis): Antiviral (valacyclovir) and corticosteroids is significantly more effective than corticosteroids alone. The medications are most effective if started within 72 hours of symptom onset. Antiviral medications alone were less effective than corticosteroids alone. This patient’s presentation is not consistent with stroke or another emergency. Because supranuclear input to the facial nerves comes from both cerebral hemispheres, strokes and other central pathologies affecting the facial nerves typically spare the forehead, which is not the case in this patient.

381
Q

Test to confirm diagnose Cushing syndrome (hypercortisolism due to many causes)?

A

24 hour urinary free cortisol level, NOT dexamethasone suppression test.

382
Q

What are risk factors for intermittent claudication?

A

Diabetes mellitus, cigarette smoking, hypertension and dyslipidemia are significant risk factors for intermittent claudication.

383
Q

Diagnostic test of choice for suspected septic arthritis is?

A

This patient has a history and physical examination concerning for septic arthritis, which is a rheumatologic emergency due to the potential for joint destruction. Joint swelling, redness, and warmth may accompany the pain but these are more difficult to detect at the hip than the knee. Systemic symptoms such as fever may occur but are absent in more than 40% of patients, particularly elderly patients and those who are immunocompromised. Risk factors for septic arthritis include underlying joint disease such as rheumatoid arthritis or osteoarthritis, and immunosuppressive states such as HIV infection, diabetes mellitus, and taking immunosuppressive medications. This patient has a history of osteoarthritis and is taking adalimumab, an immunosuppressive agent. Although there may be clues to the diagnosis of septic arthritis on imaging and laboratory assessment, the diagnostic test of choice is analysis of synovial fluid obtained through arthrocentesis. A radionuclide bone scan, CT, MR arthrography, and MRI are not sensitive enough to rule out septic arthritis.

384
Q

What is muscular dystrophy?

Signs of central cause of motor delay in children?

A

Genetic disorder of genes necessary for normal muscle function. Muscle weakness is the primary symptom.
Signs of increased muscular tone, such as cross-legged posturing, neck stiffness, and hyperreflexia, suggest a central cause of motor delay such as cerebral palsy.

385
Q

Which condition is most commonly associated with oligohydramnios?

A

Amniotic fluid volume is regulated in part by fetal swallowing, inspiration, and urination. Some malformations of the urinary tract, including renal agenesis and persistent obstruction from posterior urethral valves, lead to oliguria or anuria, and are associated with marked oligohydramnios.

Anencephaly, esophageal atresia, heart failure, and maternal α-thalassemia are associated with polyhydramnios.

386
Q

What is the most likely cause of hypercalcemia?

A

The most common cause of hypercalcemia is hyperparathyroidism. This is seldom symptomatic and is often discovered through routine blood testing. Hypercalcemia due to cancer can be caused by secretion of the parathyroid hormone–related protein and by osteoclastic bone resorption. Other causes of hypercalcemia include thiazide diuretics, lithium, vitamin D intoxication, hyperthyroidism, milk alkali syndrome from excessive calcium antacid ingestion, adrenal insufficiency, and lymphoma.

387
Q

Treatment for respiratory syncytial virus bronchiolitis in infant?

A

The mainstay of therapy for acute respiratory syncytial virus bronchiolitis is supportive care, and maintaining hydration is important. Infants with respiratory rates >60/min may have poor feeding secondary to difficulty breathing and oral rehydration may increase the risk of aspiration. In these cases, nasogastric or intravenous fluids should be administered. Oxygen saturation of 90% or more on room air is sufficient for infants with bronchiolitis, and using supplemental oxygen to maintain higher oxygen saturations only prolongs hospitalization because of an assumed need for oxygen. Bronchodilators or corticosteroids should not be administered to infants with bronchiolitis.

388
Q

Health information for patients should be written at or below what reading level?

A

It is recommended that all printed patient education information be written at or below a fifth- to
sixth-grade reading level.